Property Cases 6

39
1 In the matter of the testate estate of Emil Maurice Bachrach, deceased. MARY McDONALD BACHRACH, petitioner- appellee, vs. SOPHIE SEIFERT and ELISA ELIANOFF, oppositors-appellants. Ross, Selph, Carrascoso and Janda for appellants. Delgado and Flores for appellee. OZAETA, J.: Is a stock dividend fruit or income, which belongs to the usufructuary, or is it capital or part of the corpus of the estate, which pertains to the remainderman? That is the question raised in the appeal. The deceased E. M. Bachrach, who left no forced heir except his widow Mary McDonald Bachrach, in his last will and testament made various legacies in cash and willed the remainder of his estate as follows: Sixth: It is my will and do herewith bequeath and devise to my beloved wife Mary McDonald Bachrach for life all the fruits and usufruct of the remainder of all my estate after payment of the legacies, bequests, and gifts provided for above; and she may enjoy said usufruct and use or spend such fruits as she may in any manner wish. The will further provided that upon the death of Mary McDonald Bachrach, one-half of the all his estate "shall be divided share and share alike by and between my legal heirs, to the exclusion of my brothers." The estate of E. M. Bachrach, as owner of 108,000 shares of stock of the Atok-Big Wedge Mining Co., Inc., received from the latter 54,000 shares representing 50 per cent stock dividend on the said 108,000 shares. On June 10, 1948, Mary McDonald Bachrach, as usufructuary or life tenant of the estate, petitioned the lower court to authorize the Peoples Bank and Trust Company as administrator of the estate of E. M. Bachrach, to her the said 54,000 share of stock dividend by endorsing and delivering to her the corresponding certificate of stock, claiming that said dividend, although paid out in the form of stock, is fruit or income and therefore belonged to her as usufructuary or life tenant. Sophie Siefert and Elisa Elianoff, legal heirs of the deceased, opposed said petition on the ground that the stock dividend in question was not income but formed part of the capital and therefore belonged not to the usufructuary but to the remainderman. And they have appealed from the order granting the petition and overruling their objection. While appellants admits that a cash dividend is an income, they contend that a stock dividend is not, but merely represents an addition to the invested capital. The so-called Massachusetts rule, which prevails in certain jurisdictions in the United States, supports appellants' contention . It regards cash dividends, however large, as income, and stock dividends, however made, as capital. (Minot vs. Paine, 99 Mass., 101; 96 Am. Dec., 705.) It holds that a stock dividend is not in any true sense any true sense any dividend at all since it involves no division or severance from the corporate assets of the dividend; that it does not distribute property but simply dilutes the shares as they existed before; and that it takes nothing from the property of the corporation, and nothing to the interests of the shareholders. On the other hand, so called Pennsylvania rule, which prevails in various other jurisdictions in the United States, supports appellee's contention. This rule declares that all earnings of the corporation made prior to the death of the testator stockholder belong to the corpus of the estate, and that all earnings, when declared as dividends in whatever form, made during the lifetime of the usufructuary or life tenant. (Earp's Appeal, 28 Pa., 368.) . . . It is clear that testator intent the remaindermen should have only the corpus of the estate he left in trust, and that all dividends should go the life tenants. It is true that profits realized are not dividends until declared by the proper officials of the corporation, but distribution of profits, however made, in dividends, and the form of the distribution is immaterial. (In re Thompson's Estate, 262 Pa., 278; 105 Atl. 273, 274.) In Hite vs. Hite (93 Ky., 257; 20 S. W., 778, 780), the Court of Appeals of Kentucky, speaking thru its Chief Justice, said: . . . Where a dividend, although declared in stock, is based upon the earnings of the company, it is in reality, whether called by one name or another, the income of the capital invested in it. It is but a mode of distributing the profit. If it be not income, what is it? If it is, then it is rightfully and equitably the property of the life tenant. If it be really profit, then he should have it, whether paid in stock or money. A stock dividend proper is the issue of new shares paid for by the transfer of a sum equal to their par value from the profits and loss account to that representing capital stock; and really a corporation has no right to a dividend, either in cash or stock, except from its earnings; and a singular state of case it seems to us, an unreasonable one is presented if the company, although it rests with it whether it will declare a dividend, can bind the courts as to the proper ownership of it, and by the mode of payment substitute its will for that of that of the testator, and favor the life tenants or the remainder-men, as it may desire. It cannot, in reason, be considered that the testator contemplated such a result. The law regards substance, and not form, and such a rule might result not only in a violation of the testator's intention, but it would give the power to the corporation to beggar the life tenants, who, in this case, are the wife and children of the testator, for the benefit of the remainder-men, who may perhaps be

description

Property Cases 6

Transcript of Property Cases 6

1

In the matter of the testate estate of Emil Maurice Bachrach, deceased. MARY McDONALD BACHRACH,petitioner-

appellee,

vs.

SOPHIE SEIFERT and ELISA ELIANOFF, oppositors-appellants.

Ross, Selph, Carrascoso and Janda for appellants.

Delgado and Flores for appellee.

OZAETA, J.:

Is a stock dividend fruit or income, which belongs to the usufructuary, or is it capital or part of the corpus of the estate, which

pertains to the remainderman? That is the question raised in the appeal.

The deceased E. M. Bachrach, who left no forced heir except his widow Mary McDonald Bachrach, in his last will and testament

made various legacies in cash and willed the remainder of his estate as follows:

Sixth: It is my will and do herewith bequeath and devise to my beloved wife Mary McDonald Bachrach for life all the fruits

and usufruct of the remainder of all my estate after payment of the legacies, bequests, and gifts provided for above; and

she may enjoy said usufruct and use or spend such fruits as she may in any manner wish.

The will further provided that upon the death of Mary McDonald Bachrach, one-half of the all his estate "shall be divided share and

share alike by and between my legal heirs, to the exclusion of my brothers."

The estate of E. M. Bachrach, as owner of 108,000 shares of stock of the Atok-Big Wedge Mining Co., Inc., received from the latter 54,000 shares representing 50 per cent stock dividend on the said 108,000 shares. On June 10, 1948, Mary McDonald Bachrach, as

usufructuary or life tenant of the estate, petitioned the lower court to authorize the Peoples Bank and Trust Company as

administrator of the estate of E. M. Bachrach, to her the said 54,000 share of stock dividend by endorsing and delivering to her the

corresponding certificate of stock, claiming that said dividend, although paid out in the form of stock, is fruit or income and

therefore belonged to her as usufructuary or life tenant. Sophie Siefert and Elisa Elianoff, legal heirs of the deceased, opposed said

petition on the ground that the stock dividend in question was not income but formed part of the capital and therefore belonged not

to the usufructuary but to the remainderman. And they have appealed from the order granting the petition and overruling their

objection.

While appellants admits that a cash dividend is an income, they contend that a stock dividend is not, but merely represents an

addition to the invested capital. The so-called Massachusetts rule, which prevails in certain jurisdictions in the United States,

supports appellants' contention . It regards cash dividends, however large, as income, and stock dividends, however made, as

capital. (Minot vs. Paine, 99 Mass., 101; 96 Am. Dec., 705.) It holds that a stock dividend is not in any true sense any true sense

any dividend at all since it involves no division or severance from the corporate assets of the dividend; that it does not distribute

property but simply dilutes the shares as they existed before; and that it takes nothing from the property of the corporation, and

nothing to the interests of the shareholders.

On the other hand, so called Pennsylvania rule, which prevails in various other jurisdictions in the United States, supports appellee's

contention. This rule declares that all earnings of the corporation made prior to the death of the testator stockholder belong to the

corpus of the estate, and that all earnings, when declared as dividends in whatever form, made during the lifetime of the

usufructuary or life tenant. (Earp's Appeal, 28 Pa., 368.)

. . . It is clear that testator intent the remaindermen should have only the corpus of the estate he left in trust, and that

all dividends should go the life tenants. It is true that profits realized are not dividends until declared by the proper

officials of the corporation, but distribution of profits, however made, in dividends, and the form of the distribution is

immaterial. (In re Thompson's Estate, 262 Pa., 278; 105 Atl. 273, 274.)

In Hite vs. Hite (93 Ky., 257; 20 S. W., 778, 780), the Court of Appeals of Kentucky, speaking thru its Chief Justice, said:

. . . Where a dividend, although declared in stock, is based upon the earnings of the company, it is in reality, whether

called by one name or another, the income of the capital invested in it. It is but a mode of distributing the profit. If it be

not income, what is it? If it is, then it is rightfully and equitably the property of the life tenant. If it be really profit, then

he should have it, whether paid in stock or money. A stock dividend proper is the issue of new shares paid for by the

transfer of a sum equal to their par value from the profits and loss account to that representing capital stock; and really a

corporation has no right to a dividend, either in cash or stock, except from its earnings; and a singular state of case — it

seems to us, an unreasonable one — is presented if the company, although it rests with it whether it will declare a dividend, can bind the courts as to the proper ownership of it, and by the mode of payment substitute its will for that of

that of the testator, and favor the life tenants or the remainder-men, as it may desire. It cannot, in reason, be considered

that the testator contemplated such a result. The law regards substance, and not form, and such a rule might result not

only in a violation of the testator's intention, but it would give the power to the corporation to beggar the life tenants,

who, in this case, are the wife and children of the testator, for the benefit of the remainder-men, who may perhaps be

2

unknown to the testator, being unborn when the will was executed. We are unwilling to adopt a rule which to us seems so

arbitrary, and devoid of reason and justice. If the dividend be in fact a profit, although declared in stock, it should be held

to be income. It has been so held in Pennsylvania and many other states, and we think it the correct rule. Earp's Appeal,

28 Pa. St. 368; Cook, Stocks & S. sec. 554. . . .

We think the Pennsylvania rule is more in accord with our statutory laws than the Massachusetts rule. Under section 16 of our

Corporation Law, no corporation may make or declare any dividend except from the surplus profits arising from its business. Any

dividend, therefore, whether cash or stock, represents surplus profits. Article 471 of the Civil Code provides that the usufructuary

shall be entitled to receive all the natural, industrial, and civil fruits of the property in usufruct. And articles 474 and 475 provide as

follows:

ART. 474. Civil fruits are deemed to accrue day by day, and belong to the usufructuary in proportion to the time the

usufruct may last.

ART. 475. When a usufruct is created on the right to receive an income or periodical revenue, either in money or fruits, or

the interest on bonds or securities payable to bearer, each matured payment shall be considered as the proceeds or fruits

such right.

When it consists of the enjoyment of the benefits arising from an interest in an industrial or commercial enterprise, the

profits of which are not distributed at fixed periods, such profits shall have the same consideration.lawphil.net

In either case they shall be distributed as civil fruits, and shall be applied in accordance with the rules prescribed by the

next preceding article.

The 108,000 shares of stock are part of the property in usufruct. The 54,000 shares of stock dividend are civil fruits of the original

investment. They represent profits, and the delivery of the certificate of stock covering said dividend is equivalent to the payment

of said profits. Said shares may be sold independently of the original shares, just as the offspring of a domestic animal may be sold

independently of its mother.

The order appealed from, being in accordance with the above-quoted provisions of the Civil Code, his hereby affirmed, with costs

against the appellants.

Moran, C. J., Paras, Feria, Pablo, Bengzon, Tuason, Montemayor and Reyes, JJ., concur.

THE BACHRACH MOTOR CO., INC., plaintiff-appellee,

vs.

TALISAY-SILAY MILLING CO., ET AL., defendants-appellees.

THE PHILIPPINE NATIONAL BANK, intervenor-appellant.

Roman J. Lacson for intervenor-appellant.

Mariano Ezpeleta for plaintiff-appellee.

Nolan and Hernaez for defendants-appellees Talisay-Silay Milling Co. and Cesar Ledesma.

ROMUALDEZ, J.:

This proceeding originated in a complaint filed by the Bachrach Motor Co., Inc., against the Talisay-Silay Milling Co., Inc., for the

delivery of the amount P13,850 or promissory notes or other instruments or credit for that sum payable on June 30, 1930, as

bonus in favor of Mariano Lacson Ledesma; the complaint further prays that the sugar central be ordered to render an accounting

of the amounts it owes Mariano Lacson Ledesma by way of bonus, dividends, or otherwise, and to pay the plaintiff a sum sufficient

to satisfy the judgment mentioned in the complaint, and that the sale made by said Mariano Lacson Ledesma be declared null and

void.

The Philippine National Bank filed a third party claim alleging a preferential right to receive any amount which Mariano Lacson

Ledesma might be entitled to from the Talisay-Silay Milling Co. as bonus, because that would be civil fruits of the land mortgaged to

said bank by said debtor for the benefit of the central referred to, and by virtue of a deed of assignment, and praying that said

central be ordered to delivered directly to the intervening bank said sum on account of the latter's credit against the aforesaid

Mariano Lacson Ledesma.

The corporation Talisay-Silay Milling Co., Inc., answered the complaint stating that of Mariano Lacson Ledesma's credit, P7,500

belonged to Cesar Ledesma because he had purchased it, and praying that it be absolved from the complaint and that the proper

party be named so that the remainder might be delivered.

Cesar Ledesma, in turn, claiming to be the owner by purchase in good faith an for a reconsideration of the P7,500 which is a part of

the credit referred to above, answered praying that he be absolved from the complaint.

3

The plaintiff Bachrach Motor Co., Inc., answered the third party claim alleging that its credit against Mariano Lacson Ledesma was

prior and preferential to that of the intervening bank, and praying that the latter's complaint be dismissed.

At the trial all the parties agreed to recognize and respect the sale made in favor of Cesar Ledesma of the P7,500 part of the credit

in question, for which reason the trial court dismissed the complaint and cross-complaint against Cesar Ledesma authorizing the

defendant central to deliver to him the aforementioned sum of P7,500. And upon conclusion of the hearing, the court held that the

Bachrach Motor Co., Inc., had a preferred right to receive the amount of P11,076.02 which was Mariano Lacson Ledesma's bonus,

and it ordered the defendant central to deliver said sum to the plaintiff.

The Philippine National Bank appeals, assigning the following alleged errors as committed by the trial court:

1. In holding that the bonus which the Talisay-Silay Milling Co., Inc., bound itself to pay the planters who had mortgaged

their land to the Philippine National Bank to secure the payment of the debt of said central to said bank is not civil fruits

of said land.

2. In not holding that said bonus became subject to the mortgage executed by the defendant Mariano Lacson Ledesma to the Philippine National Bank to secure the payment of his personal debt to said bank when it fell due.

3. In holding that the assignment (Exhibit 9, P.N.B.) of said bonus made on March 7, 1930, by Mariano Lacson Ledesma

to the Philippine National Bank to be applied to the payment of his debt to said Philippine National Bank is fraudulent.

4. In holding that the Bachrach Motor Co. Inc., in civil case No. 31597 of the Court of First Instance of Manila levied a

valid attachment upon the bonus in question.

5. In admitting and considering the supplementary complaint filed by the Bachrach Motor Co., Inc., alleging as a cause of

action the attachment of the bonus in question which said Bachrach Motor Co., Inc., in civil case No. 31821 of the Court

of First Instance of Manila levied after the filing of the original complaint in this case, and after Mariano Lacson Ledesma

in this case had been declared in default.

6. In holding that the Bachrach Motor Co., Inc., has a preferential right to receive from the Talisay-Silay Milling Co., Inc.,

the amount of P11,076.02 which is in the possession of said corporation as the bonus to be paid to Mariano Lacson

Ledesma, and in ordering the Talisay-Silay Milling Co., Inc., to deliver said amount to the Bachrach Motor Co., Inc.

7. In not holding that the Philippine National Bank has a preferential right to receive from the Talisay-Silay Milling Co.,

Inc., the amount of P11,076.02 held by said corporation as Mariano Lacson Ledesma's bonus, and in not ordering said

Talisay-Silay Milling Co., Inc., to deliver said amount to the Philippine National Bank.

8. In not holding that the amended complaint and the supplementary complaint of the Bachrach Motor Co., Inc., do not

state facts sufficient to constitute a cause of action in favor of the Bachrach Motor Co., Inc., and against the Talisay-Silay

Milling Co., Inc., or against the Philippine National Bank.

The appellant bank bases its preferential right upon the contention that the bonus in question is civil fruits of the lands which the

owners had mortgaged for the benefit of the central giving the bonus, and that, as civil fruits of said land, said bonus was assigned

by Mariano Lacson Ledesma on March 7, 1930, by virtue of the document Exhibit 9 of said intervening institution, which admitted in

its brief that "if the bonus in question is not civil fruits or rent which became subject to the mortgage in favor of the Philippine

National Bank when Mariano Lacson Ledesma's personal obligation fell due, the assignment of March 7, 1930 (Exhibit 9, P.N.B.), is

null and void, not because it is fraudulent, for there was no intent of fraud in executing the deed, but that the cause or

consideration of the assignment was erroneous, for it was based upon the proposition that the bonus was civil fruits of the land mortgaged to the Philippine National Bank." (P. 31.)

The fundamental question, then, submitted to our consideration is whether or not the bonus in question is civil fruits.

This is how the bonus came to be granted: On December 22, 1923, the Talisay-Silay Milling Co., Inc., was indebted to the Philippine National Bank. To secure the payment of its debt, it succeeded in inducing its planters, among whom was Mariano Lacson Ledesma,

to mortgage their land to the creditor bank. And in order to compensate those planters for the risk they were running with their

property under the mortgage, the aforesaid central, by a resolution passed on that same date, i.e., December 22, 1923, undertook

to credit the owners of the plantation thus mortgaged every year with a sum equal to two per centum of the debt secured according

to yearly balance, the payment of the bonus being made at once, or in part from time to time, as soon as the central became free

of its obligations to the aforesaid bank, and of those contracted by virtue of the contract of supervision, and had funds which might

be so used, or as soon as it obtained from said bank authority to make such payment. (Exhibits 5, 6; P.N.B.)

Article 355 of the Civil Code considers three things as civil fruits: First, the rents of buildings; second, the proceeds from leases of

lands; and, third, the income from perpetual or life annuities, or other similar sources of revenue. It may be noted that according to

the context of the law, the phrase "u otras analogas" refers only to rent or income, for the adjectives "otras" and "analogas" agree

with the noun "rentas," as do also the other adjectives "perpetuas" and "vitalicias." That is why we say that by "civil fruits" the Civil

Code understands one of three and only three things, to wit: the rent of a building, the rent of land, and certain kinds of income.

4

As the bonus in question is not rent of a building or of land, the only meaning of "civil fruits" left to be examined is that of

"income."

Assuming that in broad juridical sense of the word "income" it might be said that the bonus in question is "income" under article

355 of the Civil Code, it is obvious to inquire whether it is derived from the land mortgaged by Mariano Lacson Ledesma to the

appellant bank for the benefit of the central; for it is not obtained from that land but from something else, it is not civil fruits of that

land, and the bank's contention is untenable.

It is to be noted that the said bonus bears no immediate, but only a remote accidental relation to the land mentioned, having been

granted as compensation for the risk of having subjected one's land to a lien in favor of the bank, for the benefit of the entity granting said bonus. If this bonus be income or civil fruits of anything, it is income arising from said risk, or, if one chooses, from

Mariano Lacson Ledesma's generosity in facing the danger for the protection of the central, but certainly it is not civil fruits or

income from the mortgaged property, which, as far as this case is concerned, has nothing to do with it. Hence, the amount of the

bonus, according to the resolution of the central granting it, is not based upon the value, importance or any other circumstance of

the mortgaged property, but upon the total value of the debt thereby secured, according to the annual balance, which is something

quite distinct from and independent of the property referred to.

Finding no merit in this appeal, the judgment appealed from is affirmed, without express finding as to costs. So ordered.

Johnson, Street, Malcolm, Villamor, Ostrand, Villa-Real, and Imperial, JJ., concur.

VICENTE STO. DOMINGO BERNARDO, Plaintiff-Appellant, vs. CATALINO BATACLAN,Defendant-Appellant.

TORIBIO TEODORO, purchaser-appellee.

Pedro de Leon for plaintiff-appellant.

Angel H. Mojica and Francisco Lavides for defendant appellant. Jose Y. Garde for appellee.

chanrobles virtual law library

LAUREL, J.: chanrobles virtual law library

This is an appeal taken by both the plaintiff and the defendant from the order of September 26, 1935, hereinabove referred to, of

the Court of First Instance of Cavite in Civil Case No. 2428.chanroblesvirtualawlibrary chanrobles virtual law library

There is no controversy as to the facts. By a contract of sale executed from Pastor Samonte and others ownership of a parcel of

land of about 90 hectares situated in sitio Balayunan, Silang, Cavite. To secure possession of the land from the vendors the said plaintiff, on July 20, 1929, instituted Civil Case No. 1935 in the Court of First Instance of Cavite. The trial court found for the

plaintiff in a decision which was affirmed by this Supreme Court on appeal (G.R. No. 33017). 1 When plaintiff entered upon the

premises, however, he found the defendant herein, Catalino Bataclan, who appears to have been authorized by former owners, as

far back as 1922, to clear the land and make improvements thereon. As Bataclan was not a party in Case No. 1935, plaintiff, on

June 11, 1931, instituted against him, in the Court of First Instance of Cavite, Civil Case No. 2428. In this case, plaintiff was

declared owner but the defendant was held to be a possessor in good faith, entitled to reimbursement in the total sum of P1,642,

for work done and improvements made. The dispositive part of the decision reads:

Por las consideraciones expuestas, se declara al demandante Vicente Santo Domingo Bernardo dueño con derecho a la posesion del

terreno que se describe en la demanda, y al demandado Catalino Bataclan con derecho a que del demandante le pague la suma de

P1,642 por gastos utiles hechos de buena fe en el terreno, y por el cerco y ponos de coco y abaca existentes en el mismo, y con

derecho, ademas a retener la posesion del terreno hasta que se le pague dicha cantidad. Al demandante puede optar, en el plazo

de treinta dias, a partir de la fecha en que fuere notificado de la presente, por pagar esa suma al demandado, haciendo asi suyos el

cerco y todas las plantaciones existentes en el terreno, u obligar al demandado a pagarle el precio terreno, a razon de trescientos

pesos la hectarea. En el caso de que el demandante optara por que el demandado le pagara el precio del terreno, el demandado

efectuara el pago en el plazo convenientes por las partes o que sera fijado por el Juzgado. Sin costas.

Both parties appealed to this court (G. R. No. 37319). 2 The decision appealed from was modified by allowing the defendant to

recover compensation amounting to P2,212 and by reducing the price at which the plaintiff could require the defendant to purchase

the land in question from P300 to P200 per hectare. Plaintiff was given by this court 30 days from the date when the decision

became final within which to exercise his option, either to sell the land to the defendant or to buy the improvements from him. On

January 9, 1934, the plaintiff manifested to the lower court his desire "to require the defendant to pay him the value of the land at

the rate of P200 per hectare or a total price of P18,000 for the whole tract of land." The defendant informed the lower court that he

was unable to pay the land and, on January 24, 1934, an order was issued giving the plaintiff 30 days within which to pay the defendant the sum of P2,212 stating that, in the event of failure to make such payment, the land would be ordered sold at public

auction " Para hacer pago al demandante de la suma de P2,212 y el remanente despues de deducidos los gastos legales de la venta

en publica subasta sera entregado al demandante." On February 21, 1934, plaintiff moved to reconsider the foregoing order so that

he would have preference over the defendant in the order of payment. The motion was denied on March 1, 1934 but on March 16

following the court below, motu proprio modified its order of January 24, " en el sentido de que el demandante tiene derecho

preferente al importe del terreno no se vendiere en publica subasta, a razon de P200 por hectares y el remanente, si acaso lo

hubiere se entregara al demandado en pago de la cantidad de P2,212 por la limpieza del terreno y las mejoras introducidas en el

5

mismo por el citado demandado." On April 24, 1934, the court below, at the instance of the plaintiff and without objection on the

part of the defendant, ordered the sale of the land in question at public auction. The land was sold on April 5, 1935 to Toribio

Teodoro, the highest bidder, for P8,000. In the certificate of sale issued to said purchaser on the very day of sale, it was stated that

the period of redemption of the land sold was to expire on April 5, 1936. Upon petition of Toribio Teodoro the court below ordered

the provincial sheriff to issue another certificate not qualified by any equity of redemption. This was complied with by the sheriff on July 30, 1935. On September 18, 1935, Teodoro moved that he be placed in possession of the land purchased by him. The motion

was granted by order of September 26, 1935, the dispositive part of which is as follows:

Por tanto, se ordena al Sheriff Provincial de Cavite ponga a Toribio Teodoro en posesion del terreno comprado por el en subasta

publica y por el cual se le expidio certificado de venta definitiva, reservando al demandado su derecho de ejercitar una accion

ordinaria para reclamar del demandante la cantidad de P2,212 a que tiene derecho por la limpieza y mejoras del terreno y cuya

suma, en justicia y equidad, debe ser descontada y deducida de la suma de P8,000 que ya ha recibido el demandante.

The Civil Code confirms certain time-honored principles of the law of property. One of these is the principle of accession whereby

the owner of property acquires not only that which it produces but that which is united to it either naturally or artificially. (Art.

353.) Whatever is built, planted or sown on the land of another, and the improvements or repairs made thereon, belong to the

owner of the land (art. 358). Where, however, the planter, builder, or sower has acted in good faith, a conflict of rights arises

between the owners and it becomes necessary to protect the owner of the improvements without causing injustice to the owner of

the land. In view of the impracticability of creating what Manresa calls a state of "forced coownership" (vol. 3, 4th ed., p. 213), the

law has provided a just and equitable solution by giving the owner of the land the option to acquire the improvements after payment of the proper indemnity or to oblige the builder or planter to pay for the land and the sower to pay the proper rent (art.

361). It is the owner of the land who is allowed to exercise the option because his right is older and because, by the principle of

accession, he is entitled to the ownership of the accessory thing (3 Manresa, 4th ed., p. 213). In the case before us, the plaintiff, as

owner of the land, chose to require the defendant, as owner of the improvements, to pay for the

land.chanroblesvirtualawlibrary chanrobles virtual law library

The defendant states that he is a possessor in good faith and that the amount of P2,212 to which he is entitled has not yet been paid to him. Therefore, he says, he has a right to retain the land in accordance with the provisions of article 453 of the Civil Code.

We do not doubt the validity of the premises stated. " Considera la ley tan saarada y legitima la deuda, que, hasta que sea pagada,

no consiente que la cosa se restituya all vencedor." (4 Manresa, 4th ed, p., 304.) We find, however, that the defendant has lost his

right of retention. In obedience to the decision of this court in G.R. No. 37319, the plaintiff expressed his desire to require the

defendant to pay for the value of the land. The said defendant could have become owner of both land and improvements and

continued in possession thereof. But he said he could not pay and the land was sold at public auction to Toribio Teodoro. The law,

as we have already said, requires no more than that the owner of the land should choose between indemnifying the owner of the

improvements or requiring the latter to pay for the land. When he failed to pay for the land, the defendant herein lost his right of

retention.chanroblesvirtualawlibrary chanrobles virtual law library

The sale at public auction having been asked by the plaintiff himself (p. 22, bill of exceptions) and the purchase price of P8,000

received by him from Toribio Teodoro, we find no reason to justify a rapture of the situation thus created between them, the

defendant-appellant not being entitled, after all, to recover from the plaintiff the sum of

P2,212.chanroblesvirtualawlibrary chanrobles virtual law library

The judgment of the lower court is accordingly modified by eliminating therefrom the reservation made in favor of the defendant-

appellant to recover from the plaintiff the sum of P2,212. In all the respects, the same is affirmed, without pronouncement

regarding costs. So ordered.chanroblesvirtualawlibrary chanrobles virtual law library

Avanceña, C.J., Villa-Real, Imperial and Diaz, JJ., concur.

DAMIAN IGNACIO, FRANCISCO IGNACIO and LUIS IGNACIO Petitioners, vs. ELIAS HILARIO and his wife DIONISIA

DRES, and FELIPE NATIVIDAD, Judge of First Instance of Pangasinan, Respondents.

Leoncio R. Esliza for petitioners.

Mauricio M. Monta for respondents.

MORAN, C.J.: chanrobles virtual law library

This is a petition for certiorari arising from a case in the Court of First Instance of Pangasinan between the herein respondents Elias

Hilario and his wife Dionisia Dres as plaintiffs, and the herein petitioners Damian, Francisco and Luis, surnamed Ignacio, as

defendants, concerning the ownership of a parcel of land, partly rice-land and partly residential. After the trial of the case, the lower

court, presided over by Hon. Alfonso Felix, rendered judgment holding plaintiffs as the legal owners of the whole property but

conceding to defendants the ownership of the houses and granaries built by them on the residential portion with the rights of a

possessor in good faith, in accordance with article 361 of the Civil Code. The dispositive part of the decision, hub of this

controversy, follows:

Wherefore, judgment is hereby rendered declaring:chanrobles virtual law library

(1) That the plaintiffs are the owners of the whole property described in transfer certificate of title No. 12872 (Exhibit A) issued in

their name, and entitled to the possession of the same;chanrobles virtual law library

6

(2) That the defendants are entitled to hold the position of the residential lot until after they are paid the actual market value of

their houses and granaries erected thereon, unless the plaintiffs prefer to sell them said residential lot, in which case defendants

shall pay the plaintiffs the proportionate value of said residential lot taking as a basis the price paid for the whole land according to

Exhibit B; and chanrobles virtual law library

(3) That upon defendant's failure to purchase the residential lot in question, said defendants shall remove their houses and

granaries after this decision becomes final and within the period of sixty (60) days from the date that the court is informed in

writing of the attitude of the parties in this respect.chanroblesvirtualawlibrary chanrobles virtual law library

No pronouncement is made as to damages and costs.chanroblesvirtualawlibrary chanrobles virtual law library

Once this decision becomes final, the plaintiffs and defendants may appear again before this court for the purpose of determining

their respective rights under article 361 of the Civil Code, if they cannot come to an extra-judicial settlement with regard to said

rights.

Subsequently, in a motion filed in the same Court of First Instance but now presided over by the herein respondent Judge Hon. Felipe Natividad, the plaintiffs prayed for an order of execution alleging that since they chose neither to pay defendants for the

buildings nor to sell to them the residential lot, said defendants should be ordered to remove the structure at their own expense

and to restore plaintiffs in the possession of said lot. Defendants objected to this motion which, after hearing, was granted by Judge

Natividad. Hence, this petition by defendants praying for (a) a restraint and annulment of the order of execution issued by Judge

Natividad; (b) an order to compel plaintiffs to pay them the sum of P2,000 for the buildings, or sell to them the residential lot for

P45; or (c), a rehearing of the case for a determination of the rights of the parties upon failure of extra-judicial

settlement.chanroblesvirtualawlibrary chanrobles virtual law library

The judgment rendered by Judge Felix is founded on articles 361 and 453 of the Civil Code which are as follows:

ART. 361. The owner of land on which anything has been built, sown or planted in good faith, shall have the right to appropriate as

his own the work, sowing or planting, after the payment of the indemnity stated in articles 453 and 454, or to oblige the one who

built or planted to pay the price of the land, and the one who sowed, the proper rent.chanroblesvirtualawlibrary chanrobles virtual

law library

ART. 453. Necessary expenses shall be refunded to every possessor; but only the possessor in good faith may retain the thing until

such expenses are made good to him.chanroblesvirtualawlibrary chanrobles virtual law library

Useful expenses shall be refunded to the possessor in good faith with the same right of retention, the person who has defeated him in the possession having the option of refunding the amount of the expenses or paying the increase in value which the thing may

have acquired in consequence thereof.

The owner of the building erected in good faith on a land owned by another, is entitled to retain the possession of the land until he

is paid the value of his building, under article 453. The owner of the land, upon the other hand, has the option, under article 361,

either to pay for the building or to sell his land to the owner of the building. But he cannot, as respondents here did, refuse both to

pay for the building and to sell the land and compel the owner of the building to remove it from the land where it is erected. He is

entitled to such remotion only when, after having chosen to sell his land, the other party fails to pay for the same. But this is not the case before us.chanroblesvirtualawlibrary chanrobles virtual law library

We hold, therefore, that the order of Judge Natividad compelling defendants-petitioners to remove their buildings from the land

belonging to plaintiffs-respondents only because the latter chose neither to pay for such buildings not to sell the land, is null and

void, for it amends substantially the judgment sought to be executed and is, furthermore, offensive to articles 361 and 453 of the

Civil Code.chanroblesvirtualawlibrary chanrobles virtual law library

There is, however, in the decision of Judge Felix a question of procedure which calls for the clarification, to avoid uncertainty and

delay in the disposition of cases. In that decision, the rights of both parties are well defined under articles 361 and 453 of the Civil

Code, but it fails to determine the value of the buildings and of the lot where they are erected as well as the periods of time within

which the option may be exercised and payment should be made, these particulars having been left for determination apparently

after the judgment has become final. This procedure is erroneous, for after the judgment has become final, no additions can be

made thereto and nothing can be done therewith except its execution. And execution cannot be had, the sheriff being ignorant as to

how, for how much, and within what time may the option be exercised, and certainly no authority is vested in him to settle these matters which involve exercise of judicial discretion. Thus the judgment rendered by Judge Felix has never become final, it having

left matters to be settled for its completion in a subsequent proceeding, matters which remained unsettled up to the time the

petition is filed in the instant case.chanroblesvirtualawlibrary chanrobles virtual law library

For all the foregoing, the writ of execution issued by Judge Natividad is hereby set aside and the lower court ordered to hold a

hearing in the principal case wherein it must determine the prices of the buildings and of the residential lot where they are erected,

as well as the period of time within which the plaintiffs-respondents may exercise their option either to pay for the buildings or to

sell their land, and, in the last instance, the period of time within which the defendants-petitioners may pay for the land, all these periods to be counted from the date the judgment becomes executory or unappealable. After such hearing, the court shall render a

final judgment according to the evidence presented by the parties.chanroblesvirtualawlibrary chanrobles virtual law library

7

The costs shall be paid by plaintiffs-respondents.chanroblesvirtualawlibrary chanrobles virtual law library

Ozaeta, Paras, Jaranilla, Feria, De Joya, Pablo, Perfecto, Hilado, Bengzon and Briones, JJ., concur.

LEONILA SARMINETO, petitioner,

vs. HON. ENRIQUE A. AGANA, District Judge, Court of First Instance of Rizal, Seventh Judicial District, Branch XXVIII,

Pasay City, and SPOUSES ERNESTO VALENTINO and REBECCA LORENZO-VALENTINO,respondents.

Mercedes M. Respicio for petitioner.

Romulo R. Bobadilla for private respondents.

MELENCIO-HERRERA, J.:ñé+.£ªwph!1

This Petition for certiorari questions a March 29, 1979 Decision rendered by the then Court of First Instance of Pasay City. The

Decision was one made on memoranda, pursuant to the provisions of RA 6031, and it modified, on October 17, 1977, a judgment

of the then Municipal Court of Paranaque, Rizal, in an Ejectment suit instituted by herein petitioner Leonila SARMIENTO against

private respondents, the spouses ERNESTO Valentino and Rebecca Lorenzo. For the facts, therefore, we have to look to the

evidence presented by the parties at the original level.

It appears that while ERNESTO was still courting his wife, the latter's mother had told him the couple could build a RESIDENTIAL

HOUSE on a lot of 145 sq. ms., being Lot D of a subdivision in Paranaque (the LAND, for short). In 1967, ERNESTO did construct a

RESIDENTIAL HOUSE on the LAND at a cost of P8,000.00 to P10,000.00. It was probably assumed that the wife's mother was the

owner of the LAND and that, eventually, it would somehow be transferred to the spouses.

It subsequently turned out that the LAND had been titled in the name of Mr. & Mrs. Jose C. Santo, Jr. who, on September 7 , 1974, sold the same to petitioner SARMIENTO. The following January 6, 1975, SARMIENTO asked ERNESTO and wife to vacate and, on

April 21, 1975, filed an Ejectment suit against them. In the evidentiary hearings before the Municipal Court, SARMIENTO submitted

the deed of sale of the LAND in her favor, which showed the price to be P15,000.00. On the other hand, ERNESTO testified that the

then cost of the RESIDENTIAL HOUSE would be from P30,000.00 to P40,000.00. The figures were not questioned by SARMIENTO.

The Municipal Court found that private respondents had built the RESIDENTIAL HOUSE in good faith, and, disregarding the

testimony of ERNESTO, that it had a value of P20,000.00. It then ordered ERNESTO and wife to vacate the LAND after SARMIENTO

has paid them the mentioned sum of P20,000.00.

The Ejectment suit was elevated to the Court of First Instance of Pasay where, after the submission of memoranda, said Court

rendered a modifying Decision under Article 448 of the Civil Code. SARMIENTO was required, within 60 days, to exercise the option

to reimburse ERNESTO and wife the sum of 40,000.00 as the value of the RESIDENTIAL HOUSE, or the option to allow them to

purchase the LAND for P25,000.00. SARMIENTO did not exercise any of the two options within the indicated period, and ERNESTO

was then allowed to deposit the sum of P25,000.00 with the Court as the purchase price for the LAND. This is the hub of the

controversy. SARMIENTO then instituted the instant certiorari proceedings.

We agree that ERNESTO and wife were builders in good faith in view of the peculiar circumstances under which they had

constructed the RESIDENTIAL HOUSE. As far as they knew, the LAND was owned by ERNESTO's mother-in-law who, having stated

they could build on the property, could reasonably be expected to later on give them the LAND.

In regards to builders in good faith, Article 448 of the Code provides:têñ.£îhqwâ£

ART. 448. The owner of the land on which anything has been built, sown or planted in good faith,

shall have the right

to appropriate as his own the works, sowing or planting, after payment of the indemnity provided for in articles

546 and 548, or

to oblige the one who built or planted to pay the price of the land, and the one who sowed, the proper rent.

However, the builder or planter cannot be obliged to buy the land if its value is considerably more than that of

the building or trees. In such case, he shall pay reasonable rent, if the owner of the land does not choose to

appropriate the building or trees after proper indemnity. The parties shall agree upon the terms of the lease

and in case of disagreement, the court shall fix the terms thereof. (Paragraphing supplied)

8

The value of the LAND, purchased for P15,000.00 on September 7, 1974, could not have been very much more than that amount

during the following January when ERNESTO and wife were asked to vacate. However, ERNESTO and wife have not questioned the

P25,000.00 valuation determined by the Court of First Instance.

In regards to the valuation of the RESIDENTIAL HOUSE, the only evidence presented was the testimony of ERNESTO that its worth

at the time of the trial should be from P30,000.00 to P40,000.00. The Municipal Court chose to assess its value at P20,000.00, or

below the minimum testified by ERNESTO, while the Court of First Instance chose the maximum of P40,000.00. In the latter case, it

cannot be said that the Court of First Instance had abused its discretion.

The challenged decision of respondent Court, based on valuations of P25,000.00 for the LAND and P40,000.00 for the RESIDENTIAL HOUSE, cannot be viewed as not supported by the evidence. The provision for the exercise by petitioner SARMIENTO of either the

option to indemnify private respondents in the amount of P40,000.00, or the option to allow private respondents to purchase the

LAND at P25,000.00, in our opinion, was a correct decision.têñ.£îhqwâ£

The owner of the building erected in good faith on a land owned by another, is entitled to retain the possession

of the land until he is paid the value of his building, under article 453 (now Article 546). The owner, of the land.

upon, the other hand, has the option, under article 361 (now Article 448), either to pay for the building or to

sell his land to the owner of the building. But he cannot, as respondents here did, refuse both to pay for the building and to sell the land and compel the owner of the building to remove it from the land where it is

erected. He is entitled to such remotion only when, after having chosen to sell his land, the other party fails to

pay for the same. (Emphasis ours)

We hold, therefore, that the order of Judge Natividad compelling defendants-petitioners to remove their

buildings from the land belonging to plaintiffs-respondents only because the latter chose neither to pay for such

buildings nor to sell the land, is null and void, for it amends substantially the judgment sought to be executed and is, furthermore, offensive to articles 361 (now Article 448) and 453 (now Article 546) of the Civil Code.

(Ignacio vs. Hilario, 76 Phil. 605, 608 [1946]).

WHEREFORE, the Petition for Certiorari is hereby ordered dismissed, without pronouncement as to costs.

SO ORDERED.1äwphï1.ñët

FRANCISCO DEPRA, plaintiff-appellee,

vs. AGUSTIN DUMLAO, defendant-appellant.

Roberto D. Dineros for plaintiff-appellee.

Veil D. Hechanova for defendant-appellant.

MELENCIO-HERRERA, J.:

This is an appeal from the Order of the former Court of First Instance of Iloilo to the then Court of Appeals, which the latter certified

to this instance as involving pure questions of law

Plaintiff-appellee, Francisco Depra, is the owner of a parcel of land registered under Transfer Certificate of Title No. T3087, known

as Lot No. 685, situated in the municipality of Dumangas, Iloilo, with an area of approximately 8,870 square meters. Agustin Dumlao, defendant-appellant, owns an adjoining lot, designated as Lot No. 683, with an approximate area of 231 sq. ms.

Sometime in 1972, when DUMLAO constructed his house on his lot, the kitchen thereof had encroached on an area of thirty four

(34) square meters of DEPRA's property, After the encroachment was discovered in a relocation survey of DEPRA's lot made on

November 2,1972, his mother, Beatriz Depra after writing a demand letter asking DUMLAO to move back from his encroachment,

filed an action for Unlawful Detainer on February 6,1973 against DUMLAO in the Municipal Court of of Dumangas, docketed as Civil

Case No 1, Said complaint was later amended to include DEPRA as a party plain. plaintiff.

After trial, the Municipal Court found that DUMLAO was a builder in good faith, and applying Article 448 of the Civil Code, rendered

judgment on September 29, 1973, the dispositive portion of which reads:

Ordering that a forced lease is created between the parties with the plaintiffs, as lessors, and the defendants as lessees, over the disputed portion with an area of thirty four (34) square meters, the rent to be paid is five

(P5.00) pesos a month, payable by the lessee to the lessors within the first five (5) days of the month the rent

is due; and the lease shall commence on the day that this decision shall have become final.

9

From the foregoing judgment, neither party appeal so that, ff it were a valid judgment, it would have ordinarily lapsed into finality,

but even then, DEPRA did not accept payment of rentals so that DUMLAO deposited such rentals with the Municipal Court.

On July 15,1974, DEPRA filed a Complaint for Quieting of Title against DUMLAO before the then Court of First Instance of Iloilo,

Branch IV (Trial Court), involving the very same 34 square meters, which was the bone of contention in the Municipal Court.

DUMLAO, in his Answer, admitted the encroachment but alleged, in the main, that the present suit is barred by res judicata by

virtue of the Decision of the Municipal Court, which had become final and executory.

After the case had been set for pre-trial, the parties submitted a Joint Motion for Judgment based on the Stipulation of Facts

attached thereto. Premised thereon, the Trial Court on October 31, 1974, issued the assailed Order, decreeing:

WHEREFORE, the Court finds and so holds that the thirty four (34) square meters subject of this litigation is

part and parcel of Lot 685 of the Cadastral Survey of Dumangas of which the plaintiff is owner as evidenced by

Transfer Certificate of Title No. 3087 and such plaintiff is entitled to possess the same.

Without pronouncement as to costs.

SO ORDERED.

Rebutting the argument of res judicata relied upon by DUMLAO, DEPRA claims that the Decision of the Municipal Court was null and

void ab initio because its jurisdiction is limited to the sole issue of possession, whereas decisions affecting lease, which is an encumbrance on real property, may only be rendered by Courts of First Instance.

Addressing out selves to the issue of validity of the Decision of the Municipal Court, we hold the same to be null and void. The

judgment in a detainer case is effective in respect of possession only (Sec. 7, Rule 70, Rules of Court). 1 The Municipal Court over-

stepped its bounds when it imposed upon the parties a situation of "forced lease", which like "forced co-ownership" is not favored in

law. Furthermore, a lease is an interest in real property, jurisdiction over which belongs to Courts of First Instance (now Regional

Trial Courts) (Sec. 44(b), Judiciary Act of 1948; 2 Sec. 19 (2) Batas Pambansa Blg. 129). 3 Since the Municipal Court, acted without jurisdiction, its Decision was null and void and cannot operate as res judicata to the subject complaint for Queting of Title. Besides,

even if the Decision were valid, the rule on res judicata would not apply due to difference in cause of action. In the Municipal Court,

the cause of action was the deprivation of possession, while in the action to quiet title, the cause of action was based on ownership.

Furthermore, Sec. 7, Rule 70 of the Rules of Court explicitly provides that judgment in a detainer case "shall not bar an action

between the same parties respecting title to the land. " 4

Conceded in the Stipulation of Facts between the parties is that DUMLAO was a builder in good faith. Thus,

8. That the subject matter in the unlawful detainer case, Civil Case No. 1, before the Municipal Court of

Dumangas, Iloilo involves the same subject matter in the present case, the Thirty-four (34) square meters

portion of land and built thereon in good faith is a portion of defendant's kitchen and has been in the

possession of the defendant since 1952 continuously up to the present; ... (Emphasis ours)

Consistent with the principle that our Court system, like any other, must be a dispute resolving mechanism, we accord legal effect

to the agreement of the parties, within the context of their mutual concession and stipulation. They have, thereby, chosen a legal

formula to resolve their dispute to appeal ply to DUMLAO the rights of a "builder in good faith" and to DEPRA those of a "landowner

in good faith" as prescribed in Article 448. Hence, we shall refrain from further examining whether the factual situations of DUMLAO

and DEPRA conform to the juridical positions respectively defined by law, for a "builder in good faith" under Article 448, a

"possessor in good faith" under Article 526 and a "landowner in good faith' under Article 448.

In regards to builders in good faith, Article 448 of the Civil Code provides:

ART. 448. The owner of the land on which anything has been built sown or planted in good faith,

shall have the right

to appropriate as his own the works, sowing or planting, after payment of the indemnity provided for in articles

546 and 548, or

to oblige the one who built or planted to pay the price of the land, and the one who sowed, the proper rent.

However, the builder or planter cannot be obliged to buy the land if its value is considerably more than that of

the building or trees. In such case, he shall pay reasonable rent, if the owner of the land does not choose to

appropriate the building or trees after proper indemnity. The parties shall agree upon the terms of the lease

and in case of disagreement, the court shall fix the terms thereof (Paragraphing supplied)

10

Pursuant to the foregoing provision, DEPRA has the option either to pay for the encroaching part of DUMLAO's kitchen, or to sell the

encroached 34 square meters of his lot to DUMLAO. He cannot refuse to pay for the encroaching part of the building, and to sell the

encroached part of his land, 5 as he had manifested before the Municipal Court. But that manifestation is not binding because it was

made in a void proceeding.

However, the good faith of DUMLAO is part of the Stipulation of Facts in the Court of First Instance. It was thus error for the Trial

Court to have ruled that DEPRA is "entitled to possession," without more, of the disputed portion implying thereby that he is

entitled to have the kitchen removed. He is entitled to such removal only when, after having chosen to sell his encroached land,

DUMLAO fails to pay for the same. 6 In this case, DUMLAO had expressed his willingness to pay for the land, but DEPRA refused to

sell.

The owner of the building erected in good faith on a land owned by another, is entitled to retain the possession

of the land until he is paid the value of his building, under article 453 (now Article 546). The owner of the land,

upon the other hand, has the option, under article 361 (now Article 448), either to pay for the building or to

sell his land to the owner of the building. But he cannot as respondents here did refuse both to pay for the

building and to sell the land and compel the owner of the building to remove it from the land where it erected.

He is entitled to such remotion only when, after having chosen to sell his land. the other party fails to pay for

the same (italics ours).

We hold, therefore, that the order of Judge Natividad compelling defendants-petitioners to remove their

buildings from the land belonging to plaintiffs-respondents only because the latter chose neither to pay for such

buildings nor to sell the land, is null and void, for it amends substantially the judgment sought to be executed

and is. furthermore, offensive to articles 361 (now Article 448) and 453 (now Article 546) of the Civil Code.

(Ignacio vs. Hilario, 76 Phil. 605, 608[1946]).

A word anent the philosophy behind Article 448 of the Civil rode.

The original provision was found in Article 361 of the Spanish Civil Code; which provided:

ART. 361. The owner of land on which anything has been built, sown or planted in good faith, shall have the right to appropriate as his own the work, sowing or planting, after the payment of the indemnity stated in

Articles 453 and 454, or to oblige the one who built or planted to pay the price of the land, and the one who

sowed, the proper rent.

As will be seen, the Article favors the owner of the land, by giving him one of the two options mentioned in the Article. Some

commentators have questioned the preference in favor of the owner of the land, but Manresa's opinion is that the Article is just and

fair.

. . . es justa la facultad que el codigo da al dueno del suelo en el articulo 361, en el caso de edificacion o

plantacion? Algunos comentaristas la conceptuan injusta, y como un extraordinario privilegio en favor de la

propiedad territorial. Entienden que impone el Codigo una pena al poseedor de buena fe y como advierte uno

de los comentaristas aludidos 'no se ve claro el por que de tal pena . . . al obligar al que obro de buena fe a

quedarse con el edificio o plantacion, previo el pago del terreno que ocupa, porque si bien es verdad que

cuando edifico o planto demostro con este hecho, que queria para si el edificio o plantio tambien lo es que el

que edifico o planto de buena fe lo hizo en la erronea inteligencia de creerse dueno del terreno Posible es que, de saber lo contrario, y de tener noticia de que habia que comprar y pagar el terreno, no se hubiera decidido a

plantar ni a edificar. La ley obligandole a hacerlo fuerza su voluntad, y la fuerza por un hecho inocente de que

no debe ser responsable'. Asi podra suceder pero la realidad es que con ese hecho voluntario, aunque sea

inocente, se ha enriquecido torticeramente con perjuicio de otro a quien es justo indemnizarle,

En nuestra opinion, el Codigo ha resuelto el conflicto de la manera mas justa y equitativa y respetando en lo

possible el principio que para la accesion se establece en el art. 358. 7

Our own Code Commission must have taken account of the objections to Article 361 of the Spanish Civil Code. Hence, the

Commission provided a modification thereof, and Article 448 of our Code has been made to provide:

ART. 448. The owner of the land on which anything has been built, sown or planted in good faith, shall have the right to appropriate as his own the works, sowing or planting, after payment of the indemnity provided for

in articles 546 and 548, or to oblige the one who built or planted to pay the price of the land, and the one who

sowed, the proper rent. However, the builder or planter cannot be obliged to buy the land if its value is

considerably more than that of the building or trees. In such case, he shall pay reasonable rent, if the owner of

the land does not choose to appropriate the building or trees after proper indemnity. The parties shall agree

upon the terms of the lease and in case of disagreement, the court shall fix the terms thereof.

Additional benefits were extended to the builder but the landowner retained his options.

The fairness of the rules in Article 448 has also been explained as follows:

11

Where the builder, planter or sower has acted in good faith, a conflict of rights arises between the owners, and

it becomes necessary to protect the owner of the improvements without causing injustice to the owner of the

land. In view of the impracticability of creating a state of forced co-ownership, the law has provided a just

solution by giving the owner of the land the option to acquire the improvements after payment of the proper

indemnity, or to oblige the builder or planter to pay for the land and the sower to pay for the proper rent. It is the owner of the land who is authorized to exercise the option, because his right is older, and because, by the

principle of accession, he is entitled to the ownership of the accessory thing. (3 Manresa 213; Bernardo vs.

Bataclan, 37 Off. Gaz. 1382; Co Tao vs. Chan Chico, G.R. No. 49167, April 30, 1949; Article applied: see

Cabral, et al vs. Ibanez [S.C.] 52 Off. Gaz. 217; Marfori vs. Velasco, [C.A.] 52 Off. Gaz. 2050). 8

WHEREFORE, the judgment of the trial Court is hereby set aside, and this case is hereby ordered remanded to the Regional Trial

Court of Iloilo for further proceedings consistent with Articles 448 and 546 of the Civil Code, as follows:

1. The trial Court shall determine

a) the present fair price of DEPRA's 34 square meter area of land;

b) the amount of the expenses spent by DUMLAO for the building of the kitchen;

c) the increase in value ("plus value") which the said area of 34 square meters may have acquired by reason

thereof, and

d) whether the value of said area of land is considerably more than that of the kitchen built thereon.

2. After said amounts shall have been determined by competent evidence, the Regional, Trial Court shall render judgment, as

follows:

a) The trial Court shall grant DEPRA a period of fifteen (15) days within which to exercise his option under the

law (Article 448, Civil Code), whether to appropriate the kitchen as his own by paying to DUMLAO either the

amount of tile expenses spent by DUMLAO f or the building of the kitchen, or the increase in value ("plus

value") which the said area of 34 square meters may have acquired by reason thereof, or to oblige DUMLAO to

pay the price of said area. The amounts to be respectively paid by DUMLAO and DEPRA, in accordance with the

option thus exercised by written notice of the other party and to the Court, shall be paid by the obligor within

fifteen (15) days from such notice of the option by tendering the amount to the Court in favor of the party

entitled to receive it;

b) The trial Court shall further order that if DEPRA exercises the option to oblige DUMLAO to pay the price of

the land but the latter rejects such purchase because, as found by the trial Court, the value of the land is

considerably more than that of the kitchen, DUMLAO shall give written notice of such rejection to DEPRA and to

the Court within fifteen (15) days from notice of DEPRA's option to sell the land. In that event, the parties shall

be given a period of fifteen (15) days from such notice of rejection within which to agree upon the terms of the

lease, and give the Court formal written notice of such agreement and its provisos. If no agreement is reached

by the parties, the trial Court, within fifteen (15) days from and after the termination of the said period fixed for negotiation, shall then fix the terms of the lease, provided that the monthly rental to be fixed by the Court

shall not be less than Ten Pesos (P10.00) per month, payable within the first five (5) days of each calendar

month. The period for the forced lease shall not be more than two (2) years, counted from the finality of the

judgment, considering the long period of time since 1952 that DUMLAO has occupied the subject area. The

rental thus fixed shall be increased by ten percent (10%) for the second year of the forced lease. DUMLAO shall

not make any further constructions or improvements on the kitchen. Upon expiration of the two-year period, or

upon default by DUMLAO in the payment of rentals for two (2) consecutive months, DEPRA shall be entitled to

terminate the forced lease, to recover his land, and to have the kitchen removed by DUMLAO or at the latter's

expense. The rentals herein provided shall be tendered by DUMLAO to the Court for payment to DEPRA, and such tender shall constitute evidence of whether or not compliance was made within the period fixed by the

Court.

c) In any event, DUMLAO shall pay DEPRA an amount computed at Ten Pesos (P10.00) per month as

reasonable compensation for the occupancy of DEPRA's land for the period counted from 1952, the year

DUMLAO occupied the subject area, up to the commencement date of the forced lease referred to in the

preceding paragraph;

d) The periods to be fixed by the trial Court in its Precision shall be inextendible, and upon failure of the party

obliged to tender to the trial Court the amount due to the obligee, the party entitled to such payment shall be

entitled to an order of execution for the enforcement of payment of the amount due and for compliance with

such other acts as may be required by the prestation due the obligee.

No costs,

12

SO ORDERED.

TECNOGAS PHILIPPINES MANUFACTURING CORPORATION, petitioner, vs.COURT OF APPEALS (FORMER SPECIAL

SEVENTEENTH DIVISION) and EDUARDO UY, respondents.

D E C I S I O N

PANGANIBAN, J.:

The parties in this case are owners of adjoining lots in Parañaque, Metro Manila. It was discovered in a survey that a portion

of a building of petitioner, which was presumably constructed by its predecessor-in-interest, encroached on a portion of the lot

owned by private respondent. What are the rights and obligations of the parties? Is petitioner considered a builder in bad faith because, as held by respondent Court, he is “presumed to know the metes and bounds of his property as described in his certif icate

of title”? Does petitioner succeed into the good faith or bad faith of his predecessor-in-interest which presumably constructed the

building?

These are the questions raised in the petition for review of the Decision[1] dated August 28, 1992, in CA-G.R. CV No. 28293 of

respondent Court[2] where the disposition reads:[3]

“WHEREFORE, premises considered, the Decision of the Regional Trial Court is hereby reversed and set aside and another one entered -

1. Dismissing the complaint for lack of cause of action;

2. Ordering Tecnogas to pay the sum of P2,000.00 per month as reasonable rental from October 4, 1979 until appellee vacates the land;

3. To remove the structures and surrounding walls on the encroached area;

4. Ordering appellee to pay the value of the land occupied by the two-storey building;

5. Ordering appellee to pay the sum of P20,000.00 for and as attorney’s fees;

6. Costs against appellee.”

Acting on the motions for reconsideration of both petitioner and private respondent, respondent Court ordered the deletion of

paragraph 4 of the dispositive portion in an Amended Decision dated February 9, 1993, as follows:[4]

“WHEREFORE, premises considered, our decision of August 28, 1992 is hereby modified deleting paragraph 4 of the dispositive

portion of our decision which reads:

‘4. Ordering appellee to pay the value of the land occupied by the two-storey building.’

The motion for reconsideration of appellee is hereby DENIED for lack of merit.”

The foregoing Amended Decision is also challenged in the instant petition.

The Facts

The facts are not disputed. Respondent Court merely reproduced the factual findings of the trial court, as follows:[5]

“That plaintiff (herein petitioner) which is a corporation duly organized and existing under and by virtue of Philippine laws is the

registered owner of a parcel of land situated in Barrio San Dionisio, Parañaque, Metro Manila known as Lot 4331-A (should be 4531-A) of Lot 4531 of the Cadastral Survey of Parañaque, Metro Manila, covered by Transfer Certificate of Title No. 409316 of the

Registry of Deeds of the Province of Rizal; that said land was purchased by plaintiff from Pariz Industries, Inc. in 1970, together

with all the buildings and improvements including the wall existing thereon; that the defendant (herein private respondent) is the

registered owner of a parcel of land known as Lot No. 4531-B of Lot 4531 of the Cadastral Survey of Parañaque, LRC (GLRO) Rec.

No. 19645 covered by Transfer Certificate of Title No. 279838, of the Registry of Deeds for the Province of Rizal; that said land

which adjoins plaintiff’s land was purchased by defendant from a certain Enrile Antonio also in 1970; that in 1971, defendant

purchased another lot also adjoining plaintiff’s land from a certain Miguel Rodriguez and the same was registered in defendant’s

name under Transfer Certificate of Title No. 31390, of the Registry of Deeds for the Province of Rizal; that portions of the buildings

13

and wall bought by plaintiff together with the land from Pariz Industries are occupying a portion of defendant’s adjoining land; that

upon learning of the encroachment or occupation by its buildings and wall of a portion of defendant’s land, plaintiff offered to buy

from defendant that particular portion of defendant’s land occupied by portions of its buildings and wall with an area of 770 square

meters, more or less, but defendant, however, refused the offer. In 1973, the parties entered into a private agreement before a

certain Col. Rosales in Malacañang, wherein plaintiff agreed to demolish the wall at the back portion of its land thus giving to defendant possession of a portion of his land previously enclosed by plaintiff’s wall; that defendant later filed a complaint before the

office of Municipal Engineer of Parañaque, Metro Manila as well as before the Office of the Provincial Fiscal of Rizal against plaintiff

in connection with the encroachment or occupation by plaintiff’s buildings and walls of a portion of its land but said complaint did

not prosper; that defendant dug or caused to be dug a canal along plaintiff’s wall, a portion of which collapsed in June, 1980, and

led to the filing by plaintiff of the supplemental complaint in the above-entitled case and a separate criminal complaint for malicious

mischief against defendant and his wife which ultimately resulted into the conviction in court of defendant’s wife for the cr ime of

malicious mischief; that while trial of the case was in progress, plaintiff filed in Court a formal proposal for settlement of the case

but said proposal, however, was ignored by defendant.”

After trial on the merits, the Regional Trial Court[6] of Pasay City, Branch 117, in Civil Case No. PQ-7631-P, rendered a

decision dated December 4, 1989 in favor of petitioner who was the plaintiff therein. The dispositive portion reads:[7]

”WHEREFORE, judgment is hereby rendered in favor of plaintiff and against defendant and ordering the latter to sell to plaint iff that

portion of land owned by him and occupied by portions of plaintiff’s buildings and wall at the price of P2,000.00 per square meter

and to pay the former:

1. The sum of P44,000.00 to compensate for the losses in materials and properties incurred by plaintiff through thievery as a result

of the destruction of its wall;

2. The sum of P7,500.00 as and by way of attorney’s fees; and

3. The costs of this suit.”

Appeal was duly interposed with respondent Court, which as previously stated, reversed and set aside the decision of the

Regional Trial Court and rendered the assailed Decision and Amended Decision. Hence, this recourse under Rule 45 of the Rules of Court.

The Issues

The petition raises the following issues:[8]

“(A)

Whether or not the respondent Court of Appeals erred in holding the petitioner a builder in bad faith because it is ‘presumed

to know the metes and bounds of his property.’

(B)

Whether or not the respondent Court of Appeals erred when it used the amicable settlement between the petitioner and the

private respondent, where both parties agreed to the demolition of the rear portion of the fence, as estoppel amounting to

recognition by petitioner of respondent’s right over his property including the portions of the land where the other structures and the building stand, which were not included in the settlement.

(C)

Whether or not the respondent Court of Appeals erred in ordering the removal of the ‘structures and surrounding walls on the encroached area’ and in withdrawing its earlier ruling in its August 28, 1992 decision for the petitioner ‘to pay for the value of the

land occupied’ by the building, only because the private respondent has ‘manifested its choice to demolish’ it despite the absence of

compulsory sale where the builder fails to pay for the land, and which ‘choice’ private respondent deliberately deleted from its

September 1, 1980 answer to the supple-mental complaint in the Regional Trial Court.”

In its Memorandum, petitioner poses the following issues:

“A

The time when to determine the good faith of the builder under Article 448 of the New Civil Code, is reckoned during the period

when it was actually being built; and in a case where no evidence was presentednor introduced as to the good faith or bad faith of

the builder at that time, as in this case, he must bepresumed to be a ‘builder in good faith,’ since ‘bad faith cannot be presumed.’[9]

14

B.

In a specific ‘boundary overlap situation’ which involves a builder in good faith, as in this case, it is now well settled that the lot owner, who builds on the adjacent lot is not charged with ‘constructive notice’ of the technical metes and bounds contained in their

torrens titles to determine the exact and precise extent of his boundary perimeter.[10]

C.

The respondent court’s citation of the twin cases of Tuason & Co. v. Lumanlan and Tuason & Co. v. Macalindong is not the ‘judicial authority’ for a boundary dispute situation between adjacent torrens titled lot owners, as the facts of the present case do not fall

within nor square with the involved principle of a dissimilar case.[11]

D.

Quite contrary to respondent Uy’s reasoning, petitioner Tecnogas continues to be a builder in good faith, even if it subsequently built/repaired the walls/other permanent structures thereon while the case a quo was pending and even while respondent sent the

petitioner many letters/filed cases thereon.[12]

D. (E.)

The amicable settlement between the parties should be interpreted as a contract and enforced only in accordance with its explicit terms, and not over and beyond that agreed upon; because the courts do nothave the power to create a contract nor expand its

scope.[13]

E. (F.)

As a general rule, although the landowner has the option to choose between: (1) ‘buying the building built in good faith’, or (2) ‘selling the portion of his land on which stands the building’ under Article 448 of the Civil Code; the first option is not absolute,

because an exception thereto, once it would be impractical for the landowner to choose to exercise the first alternative, i.e. buy

that portion of the house standing on his land, for the whole building might be rendered useless. The workable solution is for him

to select the second alternative, namely, to sell to the builder that part of his land on which was constructed a portion of the

house.”[14]

Private respondent, on the other hand, argues that the petition is “suffering from the following flaws:[15]

1. It did not give the exact citations of cases decided by the Honorable Supreme Court that allegedly contradicts the ruling of

the Hon. Court of Appeals based on the doctrine laid down in Tuason vs. Lumanlan case citing also Tuason vs.

Macalindong case (Supra).

2. Assuming that the doctrine in the alleged Co Tao vs. Chico case is contradictory to the doctrine in Tuason vs. Lumanlan

and Tuason vs. Macalindong, the two cases being more current, the same should prevail.”

Further, private respondent contends that the following “unmistakably” point to the bad faith of petitioner: (1) private respondent’s purchase of the two lots, “was ahead of the purchase by petitioner of the building and lot from Pariz Industries”; (2) the declaration

of the General Manager of Tecnogas that the sale between petitioner and Pariz Industries “was not registered” because of some

problems with China Banking Corporation; and (3) the Deed of Sale in favor of petitioner was registered in its name only in “the

month of May 1973.”[16]

The Court’s Ruling

The petition should be granted.

Good Faith or Bad Faith

Respondent Court, citing the cases of J. M. Tuason & Co., Inc. vs. Vda. de Lumanlan[17] and J. M. Tuason & Co., Inc. vs.

Macalindong,[18] ruled that petitioner “cannot be considered in good faith” because as a land owner, it is “presumed to know the

metes and bounds of his own property, specially if the same are reflected in a properly issued certificate of title. One who erroneously builds on the adjoining lot should be considered a builder in (b)ad (f)aith, there being presumptive knowledge of the

Torrens title, the area, and the extent of the boundaries.”[19]

15

We disagree with respondent Court. The two cases it relied upon do not support its main pronouncement that a registered

owner of land has presumptive knowledge of the metes and bounds of its own land, and is therefore in bad faith if he mistakenly

builds on an adjoining land. Aside from the fact that those cases had factual moorings radically different from those obtaining here,

there is nothing in those cases which would suggest, however remotely, that bad faith is imputable to a registered owner of land

when a part of his building encroaches upon a neighbor’s land, simply because he is supposedly presumed to know the boundaries of his land as described in his certificate of title. No such doctrinal statement could have been made in those cases because such

issue was not before the Supreme Court. Quite the contrary, we have rejected such a theory in Co Tao vs. Chico,[20] where we held

that unless one is versed in the science of surveying, “no one can determine the precise extent or location of his property by merely

examining his paper title.”

There is no question that when petitioner purchased the land from Pariz Industries, the buildings and other structures were

already in existence. The record is not clear as to who actually built those structures, but it may well be assumed that petitioner’s

predecessor-in-interest, Pariz Industries, did so. Article 527 of the Civil Code presumes good faith, and since no proof exists to

show that the encroachment over a narrow, needle-shaped portion of private respondent’s land was done in bad faith by the builder

of the encroaching structures, the latter should be presumed to have built them in good faith.[21] It is presumed that possession

continues to be enjoyed in the same character in which it was acquired, until the contrary is proved.[22] Good faith consists in the belief of the builder that the land he is building on is his, and his ignorance of any defect or flaw in his title.[23] Hence, such good

faith, by law, passed on to Pariz’s successor, petitioner in this case. Further, “(w)here one derives title to property from another,

the act, declaration, or omission of the latter, while holding the title, in relation to the property, is evidence against the

former.”[24] And possession acquired in good faith does not lose this character except in case and from the moment facts exist

which show that the possessor is not unaware that he possesses the thing improperly or wrongfully.[25] The good faith ceases from

the moment defects in the title are made known to the possessor, by extraneous evidence or by suit for recovery of the property by

the true owner.[26]

Recall that the encroachment in the present case was caused by a very slight deviation of the erected wall (as fence) which

was supposed to run in a straight line from point 9 to point 1 of petitioner’s lot. It was an error which, in the context of the attendant facts, was consistent with good faith. Consequently, the builder, if sued by the aggrieved landowner for recovery of

possession, could have invoked the provisions of Art. 448 of the Civil Code, which reads:

”The owner of the land on which anything has been built, sown or planted in good faith, shall have the right to appropriate as his

own the works, sowing or planting, after payment of the indemnity provided for in articles 546 and 548, or to oblige the one who

built or planted to pay the price of the land, and the one who sowed, the proper rent. However, the builder or planter cannot be

obliged to buy the land if its value is considerably more than that of the building or trees. In such case, he shall pay reasonable rent, if the owner of the land does not choose to appropriate the building or trees after proper indemnity. The parties shall agree

upon the terms of the lease and in case of disagreement, the court shall fix the terms thereof.”

The obvious benefit to the builder under this article is that, instead of being outrightly ejected from the land, he can compel the

landowner to make a choice between the two options: (1) to appropriate the building by paying the indemnity required by law, or

(2) sell the land to the builder. The landowner cannot refuse to exercise either option and compel instead the owner of the building

to remove it from the land.[27]

The question, however, is whether the same benefit can be invoked by petitioner who, as earlier stated, is not the builder of the offending structures but possesses them as buyer.

We answer such question in the affirmative.

In the first place, there is no sufficient showing that petitioner was aware of the encroachment at the time it acquired the

property from Pariz Industries. We agree with the trial court that various factors in evidence adequately show petitioner’s lack of

awareness thereof. In any case, contrary proof has not overthrown the presumption of good faith under Article 527 of the Civil

Code, as already stated, taken together with the disputable presumptions of the law on evidence. These presumptions state, under

Section 3 (a) of Rule 131 of the Rules of Court, that the person is innocent of a crime or wrong; and under Section 3 (ff) of Rule

131, that the law has been obeyed. In fact, private respondent Eduardo Uy himself was unaware of such intrusion into his property

until after 1971 when he hired a surveyor, following his purchase of another adjoining lot, to survey all his newly acquired lots. Upon being apprised of the encroachment, petitioner immediately offered to buy the area occupied by its building -- a species

of conduct consistent with good faith.

In the second place, upon delivery of the property by Pariz Industries, as seller, to the petitioner, as buyer, the latter

acquired ownership of the property. Consequently and as earlier discussed, petitioner is deemed to have stepped into the shoes of

the seller in regard to all rights of ownership over the immovable sold, including the right to compel the private respondent to

exercise either of the two options provided under Article 448 of the Civil Code.

Estoppel

Respondent Court ruled that the amicable settlement entered into between petitioner and private respondent estops the

former from questioning the private respondent’s “right” over the disputed property. It held that by undertaking to demolish the

fence under said settlement, petitioner recognized private respondent’s right over the property, and “cannot later on compel” private respondent “to sell to it the land since” private respondent “is under no obligation to sell.”[28]

We do not agree. Petitioner cannot be held in estoppel for entering into the amicable settlement, the pertinent portions of

which read:[29]

16

”That the parties hereto have agreed that the rear portion of the fence that separates the property of the complainant and

respondent shall be demolished up to the back of the building housing the machineries which demolision (sic) shall be undertaken

by the complainant at anytime.

That the fence which serve(s) as a wall housing the electroplating machineries shall not be demolished in the mean time which

portion shall be subject to negotiation by herein parties.”

From the foregoing, it is clear that petitioner agreed only to the demolition of a portion of the wall separating the adjoining

properties of the parties -- i.e. “up to the back of the building housing the machineries.” But that portion of the fence which served

as the wall housing the electroplating machineries was not to be demolished. Rather, it was to “be subject to negotiation by herein parties.” The settlement may have recognized the ownership of private respondent but such admission cannot be equated with bad

faith. Petitioner was only trying to avoid a litigation, one reason for entering into an amicable settlement.

As was ruled in Osmeña vs. Commission on Audit,[30]

“A compromise is a bilateral act or transaction that is expressly acknowledged as a juridical agreement by the Civil Code and is

therein dealt with in some detail. `A compromise,’ declares Article 2208 of said Code, `is a contract whereby the parties, by

making reciprocal concessions, avoid a litigation or put an end to one already commenced.’

xxx xxx xxx

The Civil Code not only defines and authorizes compromises, it in fact encourages them in civil actions. Art. 2029 states that `The

Court shall endeavor to persuade the litigants in a civil case to agree upon some fair compromise.’ x x x.”

In the context of the established facts, we hold that petitioner did not lose its rights under Article 448 of the Civil Code on the

basis merely of the fact that some years after acquiring the property in good faith, it learned about -- and aptly recognized -- the

right of private respondent to a portion of the land occupied by its building. The supervening awareness of the encroachment by

petitioner does not militate against its right to claim the status of a builder in good faith. In fact, a judicious reading of said Article

448 will readily show that the landowner’s exercise of his option can only take place after the builder shall have come to know of

the intrusion -- in short, when both parties shall have become aware of it. Only then will the occasion for exercising the option

arise, for it is only then that both parties will have been aware that a problem exists in regard to their property rights.

Options of Private Respondent

What then is the applicable provision in this case which private respondent may invoke as his

remedy: Article 448 or Article 450[31] of the Civil Code?

In view of the good faith of both petitioner and private respondent, their rights and obligations are to be governed by Art.

448. The essential fairness of this codal provision has been pointed out by Mme. Justice Ameurfina Melencio-Herrera, citing

Manresa and applicable precedents, in the case of Depra vs. Dumlao,[32] to wit:

“Where the builder, planter or sower has acted in good faith, a conflict of rights arises between the owners, and it becomes necessary to protect the owner of the improvements without causing injustice to the owner of the land. In view of the

impracticality of creating a state of forced co-ownership, the law has provided a just solution by giving the owner of the land the

option to acquire the improvements after payment of the proper indemnity, or to oblige the builder or planter to pay for the land

and the sower to pay the proper rent. It is the owner of the land who is authorized to exercise the option, because his right is older,

and because, by the principle of accession, he is entitled to the ownership of the accessory thing. (3 Manresa 213; Bernardo vs.

Bataclan, 37 Off. Gaz. 1382; Co Tao vs. Chan Chico, G. R. No. 49167, April 30, 1949; Article applied; see Cabral, et al. vs. Ibanez

[S.C.] 52 Off. Gaz. 217; Marfori vs. Velasco, [C.A.] 52 Off. Gaz. 2050).”

The private respondent’s insistence on the removal of the encroaching structures as the proper remedy, which respondent

Court sustained in its assailed Decisions, is thus legally flawed. This is not one of the remedies bestowed upon him by law. It would

be available only if and when he chooses to compel the petitioner to buy the land at a reasonable price but the latter fails to pay

such price.[33] This has not taken place. Hence, his options are limited to: (1) appropriating the encroaching portion of petitioner’s

building after payment of proper indemnity, or (2) obliging the latter to buy the lot occupied by the structure. He cannot exercise a

remedy of his own liking.

Neither is petitioner’s prayer that private respondent be ordered to sell the land[34] the proper remedy. While that was

dubbed as the “more workable solution” in Grana and Torralba vs. The Court of Appeals, et al.,[35] it was not the relief granted in that case as the landowners were directed to exercise “within 30 days from this decision their option to either buy the portion of the

petitioners’ house on their land or sell to said petitioners the portion of their land on which it stands.”[36] Moreover, in Grana and

Torralba, the area involved was only 87 square meters while this case involves 520 square meters[37]. In line with the case of

Depra vs. Dumlao,[38] this case will have to be remanded to the trial court for further proceedings to fully implement the mandate of

Art. 448. It is a rule of procedure for the Supreme Court to strive to settle the entire controversy in a single proceeding leaving no

root or branch to bear the seeds of future litigation.[39]

17

Petitioner, however, must also pay the rent for the property occupied by its building as prescribed by respondent Court from

October 4, 1979, but only up to the date private respondent serves notice of its option upon petitioner and the trial court; that is, if

such option is for private respondent to appropriate the encroaching structure. In such event, petitioner would have a right of

retention which negates the obligation to pay rent.[40] The rent should however continue if the option chosen is compulsory sale, but

only up to the actual transfer of ownership.

The award of attorney’s fees by respondent Court against petitioner is unwarranted since the action appears to have been

filed in good faith. Besides, there should be no penalty on the right to litigate.[41]

WHEREFORE, premises considered, the petition is hereby GRANTED and the assailed Decision and the Amended Decision are

REVERSED and SET ASIDE. In accordance with the case of Depra vs. Dumlao,[42] this case is REMANDED to the Regional Trial Court

of Pasay City, Branch 117, for further proceedings consistent with Articles 448 and 546 [43] of the Civil Code, as follows:

The trial court shall determine:

a) the present fair price of private respondent’s 520 square-meter area of land;

b) the increase in value (“plus value”) which the said area of 520 square meters may have acquired by reason of

the existence of the portion of the building on the area;

c) the fair market value of the encroaching portion of the building; and

d) whether the value of said area of land is considerably more than the fair market value of the portion of the

building thereon.

2. After said amounts shall have been determined by competent evidence, the regional trial court shall render judgment as follows:

a) The private respondent shall be granted a period of fifteen (15) days within which to exercise his option under the

law (Article 448, Civil Code), whether to appropriate the portion of the building as his own by paying to petitioner

its fair market value, or to oblige petitioner to pay the price of said area. The amounts to be respectively paid by petitioner and private respondent, in accordance with the option thus exercised by written notice of the other party

and to the court, shall be paid by the obligor within fifteen (15) days from such notice of the option by tendering

the amount to the trial court in favor of the party entitled to receive it;

b) If private respondent exercises the option to oblige petitioner to pay the price of the land but the latter rejects such

purchase because, as found by the trial court, the value of the land is considerably more than that of the portion of

the building, petitioner shall give written notice of such rejection to private respondent and to the trial court within

fifteen (15) days from notice of private respondent’s option to sell the land. In that event, the parties shall be

given a period of fifteen (15) days from such notice of rejection within which to agree upon the terms of the lease,

and give the trial court formal written notice of the agreement and its provisos. If no agreement is reached by the parties, the trial court, within fifteen (15) days from and after the termination of the said period fixed for

negotiation, shall then fix the terms of the lease provided that the monthly rental to be fixed by the Court shall not

be less than two thousand pesos (P2,000.00) per month, payable within the first five (5) days of each calendar

month. The period for the forced lease shall not be more than two (2) years, counted from the finality of the

judgment, considering the long period of time since 1970 that petitioner has occupied the subject area. The rental

thus fixed shall be increased by ten percent (10%) for the second year of the forced lease. Petitioner shall not

make any further constructions or improvements on the building. Upon expiration of the two-year period, or upon

default by petitioner in the payment of rentals for two (2) consecutive months, private respondent shall be entitled

to terminate the forced lease, to recover his land, and to have the portion of the building removed by petitioner or

at latter’s expense. The rentals herein provided shall be tendered by petitioner to the trial court for payment to private respondent, and such tender shall constitute evidence of whether or not compliance was made within the

period fixed by the said court.

c) In any event, petitioner shall pay private respondent an amount computed at two thousand pesos (P2,000.00) per

month as reasonable compensation for the occupancy of private respondent’s land for the period counted from

October 4, 1979, up to the date private respondent serves notice of its option to appropriate the encroaching

structures, otherwise up to the actual transfer of ownership to petitioner or, in case a forced lease has to be

imposed, up to the commencement date of the forced lease referred to in the preceding paragraph;

d) The periods to be fixed by the trial court in its decision shall be non-extendible, and upon failure of the party obliged

to tender to the trial court the amount due to the obligee, the party entitled to such payment shall be entitled to an order of execution for the enforcement of payment of the amount due and for compliance with such other acts as

may be required by the prestation due the obligee.

No costs.

SO ORDERED.

BARTOLOME ORTIZ, petitioner, vs.

18

HON. UNION C. KAYANAN, in his capacity as Judge of the Court of First Instance of Quezon, Branch IV; ELEUTERIO

ZAMORA, QUIRINO COMINTAN, VICENTE FERRO, AND GREGORIO PAMISARAN,respondents.

Salonga, Ordoñ;ez, Yap, Sicat & Associates and Salvador, Ulgado & Carbon for petitioner.

Jose A. Cusi for private respondents.

ANTONIO, J.:1äwphï1.ñët

Petition for certiorari and Prohibition with Preliminary Injunction to nullify the Order of respondent Judge directing the execution of

the final judgment in Civil Case No. C-90, entitled "Bartolome Ortiz vs. Secretary of Agriculture and Natural Resources, et al.," and the Writ of Execution issued to implement said Order, allegedly for being inconsistent with the judgment sought to be enforced.

Civil Case No. C-90 was filed by Bartolome Ortiz who sought the review and/or annulment of the decision of the Secretary of

Agriculture and Natural Resources, giving preference to the sales applications of private respondents Quirino Comintan and

Eleuterio Zamora over Lot No. 5785, PLS-45, located at Barrio Cabuluan, Calauag, Quezon.

I

The factual background of the case, as found by respondent Court, is as follows:têñ.£îhqwâ£

... The lot in controversy was formerly the subject of Homestead Application No. 122417 of Martin Dolorico II, plaintiff's ward who died on August 20, 1931; that since then it was plaintiff who continued the cultivation and

possession of the property, without however filing any application to acquire title thereon; that in the

Homestead Application No. 122417, Martin Dolorico II named his uncle, Martin Dolorico I as his heir and

successor in interest, so that in 1951 Martin Dolorico I executed an affidavit relinquishing his rights over the

property in favor of defendants Quirino Comintan and Eleuterio Zamora, his grandson and son-in-law,

respectively, and requested the Director of Lands to cancel the homestead application; that on the strength of

the affidavit, Homestead Application No. 122417 was cancelled and thereafter, defendants Comintan and

Zamora filed their respective sales applications Nos. 8433 and 9258; that plaintiff filed his protest on November

26, 1951 alleging that he should be given preference to purchase the lot inasmuch as he is the actual occupant and has been in continuous possession of the same since 1931; and inspite of plaintiff's opposition, "Portion A"

of the property was sold at public auction wherein defendant Comintan was the only bidder; that on June 8,

1957, investigation was conducted on plaintiff's protest by Assistant Public Lands Inspector Serapion Bauzon

who submitted his report to the Regional Land Officer, and who in turn rendered a decision on April 9, 1958,

dismissing plaintiff's claim and giving due course to defendants' sales applications on the ground that the

relinquishment of the homestead rights of Martin Dolorico I in favor of Comintan and Zamora is proper, the

former having been designated as successor in interest of the original homestead applicant and that because

plaintiff failed to participate in the public auction, he is forever barred to claim the property; that plaintiff filed a

motion for reconsideration of this decision which was denied by the Director of Lands in his order dated June

10, 1959; that, finally, on appeal to the Secretary of Agriculture and Natural Resources, the decision rendered by the Regional Land Officer was affirmed in toto. 1

On March 22, 1966, respondent Court rendered judgment in the afore-mentioned civil case, the dispositive portion of which reads

as follows:têñ.£îhqwâ£

IN VIEW OF THE FOREGOING CONSIDERATIONS, judgment is hereby rendered awarding Lot No. 5785-A of

PLS-45, (Calauag Public Land Subdivision) one-half portion of the property in litigation located at Bo. Cabuluan,

Calauag, Quezon, in favor of defendant QUIRINO COMINTAN, being the successful bidder in the public auction

conducted by the bureau of Lands on April 18, 1955, and hereby giving due course to the Sales Application No.

9258 of defendant Eleuterio Zamora over the other half, Lot No. 5785-B of PLS-45, Calauag, without prejudice

to the right of plaintiff BARTOLOME ORTIZ to participate in the public bidding of the same to be announced by

the Bureau of Lands, Manila. However, should plaintiff Bartolome Ortiz be not declared the successful bidder

thereof, defendants Quirino Comintan and Eleuterio Zamora are ordered to reimburse jointly said plaintiff the

improvements he has introduced on the whole property in the amount of THIRTEEN THOUSAND SIX HUNDRED THIRTY-TWO (P13,632.00) PESOS, the latter having the right to retain the property until after he has been

fully paid therefor, without interest since he enjoys the fruits of the property in question, with prejudice and

with costs again the plaintiff. 2

Plaintiff appealed the decision to the Court of Appeals.

Two (2) years after the rendition of the judgment by the court a quo, while the case was pending appeal and upon petition of

private respondents Quirino Comintan and Eleuterio Zamora, respondent Court appointed respondent Vicente Ferro, Clerk of Court,

as Receiver to collect tolls on a portion of the property used as a diversion road. On August 19, 1969, the Court of Appeals issued a

Resolution annulling the Order appointing the Receiver. Subsequently, on February 19, 1970, the Appellate Court affirmed the

19

decision of the trial court. A petition for review on certiorari of the decision of the Court of Appeals was denied by this Court on April

6, 1970. At this point, private respondents filed a petition for appointment of a new receiver with the court a quo. This petition was

granted and the receiver was reappointed. Petitioner sought the annulment of this Order with the Court of Appeals, but said Court

ruled that its decision had already become final and that the records of the case were to be remanded to the trial court.

Not satisfied with such denial, petitioner filed a petitioner for certiorari, prohibition and mandamus with preliminary injunction

before this Court, 3 praying for the annulment of the Order reappointing the Receiver. On July 13, 1970, the petition was dismissed

by this Court on the ground of insufficient showing of grave abuse of discretion.

II

The judgment having become final and executory private respondents filed a motion for the execution of the same, praying as

follows:têñ.£îhqwâ£

WHEREFORE, it is respectfully prayed of this Honorable Court to order the issuance of a writ of execution in

accordance with the judgment of this Honorable Court, confirmed by the Court of Appeals and the Supreme Court, commanding any lawful officer to deliver to defendants Comintan and Zamora the land subject of the

decision in this case but allowing defendants to file a bond in such amount as this Honorable Court may fix, in

lieu of the P13,632.00 required to be paid to plaintiff, conditioned that after the accounting of the tools

collected by plaintiff, there is still an amount due and payable to said plaintiff, then if such amount is not paid

on demand, including the legal interests, said bond shall be held answerable.

Ordering further the plaintiff to render an accounting of the tolls he collected from March of 1967 to December 31, 1968 and from September 1969 to March 31, 1970, and deliver said tolls collected to the receiver and if

judgment is already executed, then to Quirino Comintan and Eleuterio Zamora; and,

Finally, to condemn plaintiff to pay moral damages for withholding the tools which belong to your movant in an

amount this Court may deem just in the premises. 4

Acting upon the foregoing motion, respondent Judge issued an Order, dated September 23, 1970, stating, among others, the

following: têñ.£îhqwâ£

The records further disclosed that from March 1967 to December 31, 1968, piaintiff Bartolome Ortiz collected

tolls on a portion of the propertv in question wherein he has not introduced anv improvement particularlv on

Lot No. 5785-A; PLS-45 awarded to defendant Quirino Comintan, thru which vehicular traffic was detoured or diverted, and again from September 1969 to March 31, 1970, the plaintiff resumed the collection of tools on

the same portion without rendering any accounting on said tolls to the Receiver, who, was reappointed after

submitting the required bond and specifically authorized only to collect tolls leaving the harvesting of the

improvements to the plaintiff.

xxx xxx xxx

ln virtue of he findings of this Court as contained in the dispositive portion of its decision, the defendants are

jointly obligated to pay the plaintiff in the amount of P13,632.00 as reasonable value of the improvements he

introduced on the whole property in question, and that he has the right of retention until fully paid. It can be

gleaned from the motion of the defendants that if plaintiff submits an accounting of the tolls he collected during

the periods above alluded to, their damages of about P25,000.00 can more than offset their obligation of

P13,362.00 in favor of the plaintiff, thereafter the possession of the land be delivered to the defendants since

the decision of the Supreme Court has already become final and executory, but in the interregnum pending

such accounting and recovery by the Receiver of the tolls collected by the plaintiff, the defendants pray that they allowed to put up a bond in lieu of the said P13,632.00 to answer for damages of the former, if any.

On the other hand, plaintiff contends in his opposition, admitting that the decision of the Supreme Court has

become final and executory; (1) the offer of a bond in lieu of payment of P13,632.00 does not, and cannot,

satisfy the condition imposed in the decision of this Court which was affirmed in toto; (2) the public sale of

Portion "B" of the land has still to take place as ordained before the decision could be executed; and, (3) that

whatever sums plaintiff may derive from the property cannot be set off against what is due him for the improvements he made, for which he has to be reimbursed as ordered.

xxx xxx xxx

Let it be known that plaintiff does not dispute his having collected tolls during the periods from March 1967 to December 31, 1968 and from September 1969 to March 31, 1970. The Supreme Court affirmed the decision of

this Court its findings that said tolls belong to the defendant, considering that the same were collected on a

portion of the land question where the plaintiff did notintroduce any improvement. The reimbursement to the

plaintiff pertains only to the value of the improvements, like coconut trees and other plants which he

introduced on the whole property. The tolls collected by the plaintiff on an unimproved portion naturally belong

20

to the defendants, following the doctrine on accretion. Further, the reappointment of a Receiver by this Court

was upheld by the Supreme Court when it denied the petition for certiorari filed by the plaintiff, bolstering the

legal claim of defendants over said tolls. Thus, the decision of the Supreme Court rendered the decision of this

Court retroactive from March 22, 1966 although pending accounting of the tolls collected by the plaintiff is

justified and will not prejudice anybody, but certainly would substantially satisfy the conditions imposed in the decision. However, insofar as the one-half portion "B" of the property, the decision may be executed only after

public sale by the Bureau of Lands shall be accomplished.

WHEREFORE, finding the Motion for Execution filed by the defendants to be meritorious, the same is granted;

provided, however, that they put up a bond equal the adjudicated amount of P13,632.00 accruing in favor of

the plaintiff, from a reputable or recognized bonding or surety company, conditioned that after an accounting of

the tolls collected by the plaintiff should there be found out any balance due and payable to him after reckoning

said obligation of P13,632.00 the bond shall be held answerable therefor. 5

Accordingly, a Writ of Execution was issued after private respondent Quirino Comintan had filed the required bond. The writ

directed the Sheriff to enforce the decision of the Court, and stated, part in, the following:têñ.£îhqwâ£

But should there be found any amount collectible after accounting and deducting the amount of P3,632.00, you are hereby ordered that of the goods and chattels of Bartolome Ortiz of Bo. Kabuluan, Calauag, Quezon, be

caused to be made any excess in the above-metioned amount together with your lawful fees and that you

render same to defendant Quirino Comintan. If sufficient personal property cannot be found thereof to satisfy

this execution and lawful fees thereon, then you are commanded that of the lands and buildings of the said

BARTOLOME ORTIZ you make the said excess amount in the manner required by the Rules of Court, and make

return of your proceedings within this Court within sixty (60) days from date of service.

You are also ordered to cause Bartolome Ortiz to vacate the property within fifteen (15) days after service

thereof the defendant Quirino Comintan having filed the required bond in the amount of THIRTEEN THOUSAND

SIX HUNDRED THIRTY-TWO (P13,632.00) PESOS. 6

On October 12, 1970, petitioner filed a Motion for Reconsideration of the aforesaid Order and Writ of Execution,

alleging:têñ.£îhqwâ£

(a) That the respondent judge has no authority to place respondents in possession of the property;

(b) That the Supreme Court has never affirmed any decision of the trial court that tolls collected from the

diversionary road on the property, which is public land, belong to said respondents;

(c) That to assess petitioner a P25,000.00 liability for damages is purely punitive imposition without factual or

legal justification.

The foregoing Motion for Reconsideration was denied by respondent Judge per Order dated November 18, 1970. Saod Order states,

in part:têñ.£îhqwâ£

It goes without saying that defendant Comintan is entitled to be placed in possession of lot No. 5785-A of PLS-

45 (Calauag Public Land Subdivision) and enjoyment of the tolls from March, 1967 to March, 1968 and from

September, 1969 to March 31, l970 which were received by plaintiff Bartolome Ortiz, collected from the

property by reason of the diversion road where vehicular traffic was detoured. To defendant Comintan belongs

the tolls thus collected from a portion of the land awarded to him used as a diversionary road by the doctrine of

accretion and his right over the same is ipso jure, there being no need of any action to possess said addition. It

is so because as consistently maintained by the Supreme Court, an applicant who has complied with all the terms and conditions which entitle him to a patent for a particular tract of publlic land, acquires a vested right

therein and is to be regarded as equitable owner thereof so that even without a patent, a perfected homestead

or sales application is a property right in the fullest sense, unaffectcd by the fact that the paramount title is still

in the Government and no subsequent law can deprive him of that vested right The question of the actual

damages suffered by defendant Comintan by reason of the unaccounted tolls received by plaintiff had already

been fully discussed in the order of September 23, 1970 and the Court is honestly convinced and believes it to

be proper and regular under the circumstances.

Incidentally, the Court stands to correct itself when in the same order, it directed the execution of he decision

with respect to the one-half portion "B" of the property only after the public sale by the Bureau of Lands, the

same being an oversight, it appearing that the Sales Application of defendant Eleuterio Zamora had already

been recognized and full confirmed by the Supreme Court.

In view thereof, finding the motion filed by plaintiff to be without merit, the Court hereby denies the same and

the order of September 23, 1970 shall remain in full force subject to the amendment that the execution of the decision with respect to the one-half portion "B" shall not be conditioned to the public sale by the Bureau of

Lands.

21

SO ORDERED. 7

III

Petitioner thus filed the instant petition, contending that in having issued the Order and Writ of Execution, respondent Court "acted

without or in excess of jurisdiction, and/or with grave abuse of discretion, because the said order and writ in effect vary the terms

of the judgment they purportedly seek to enforce." He argued that since said judgment declared the petitioner a possessor in good

faith, he is entitled to the payment of the value of the improvements introduced by him on the whole property, with right to retain

the land until he has been fully paid such value. He likewise averred that no payment for improvements has been made and,

instead, a bond therefor had been filed by defendants (private respondents), which, according to petitioner, is not the payment envisaged in the decision which would entitle private respondents to the possession of the property. Furthermore, with respect to

portion "B", petitioner alleges that, under the decision, he has the right to retain the same until after he has participated and lost in

the public bidding of the land to be conducted by the Bureau of Lands. It is claimed that it is only in the event that he loses in the

bidding that he can be legally dispossessed thereof.

It is the position of petitioner that all the fruits of the property, including the tolls collected by him from the passing vehicles, which

according to the trial court amounts to P25,000.00, belongs to petitioner and not to defendant/private respondent Quirino

Comintan, in accordance with the decision itself, which decreed that the fruits of the property shall be in lieu of interest on the amount to be paid to petitioner as reimbursement for improvements. Any contrary opinion, in his view, would be tantamount to an

amendment of a decision which has long become final and executory and, therefore, cannot be lawfully done.

Petitioner, therefore, prayed that: (1) a Writ of Preliminary Injunction be issued enjoining the enforcement of the Orders of

September 23, 1970 and November 18, 1970, and the Writ of Execution issued thereto, or restoring to petitioner the possession of

the property if the private respondents had been placed in possession thereof; (2) annulling said Orders as well as the Writ of

Execution, dissolving the receivership established over the property; and (3) ordering private respondents to account to petitioner all the fruits they may have gathered or collected from the property in question from the time of petitioiier's illegal dispossession

thereof.

On January 29, 1971, this Court issued the Writ of Preliminary Injunction. On January 30, 1971, private respondents filed a Motion

for Reconsideration and/or Modification of the Order dated January 29, 1971. This was followed by a Supplemental Motion for

Reconsideration and Manifestation on February 3, 1971. In the latter motion, private respondents manifested that the amount of

P14,040.96, representing the amount decreed in the judgment as reimbursement to petitioner for the improvements, plus interest for six months, has already been deposited by them in court, "with the understanding that said amount shall be turned over to the

plaintiff after the court a quo shall have determined the improvement on Lot 5785-A, and subsequently the remaining balance of

the deposit shall be delivered to the petitioner (plaintiff therein) in the event he loses the bid for Lot 5785-B in favor of private

respondent Eleuterio Zamora." 8 The deposit is evidenced by a certification made by the Clerk of the Court a quo. 9 Contending that

said deposit was a faithful compliance with the judgment of the trial court, private respondent Quirino Comintan prayed for the

dissolution of the Writ of Injunction.

It appears that as a consequence of the deposit made by private respondents, the Deputy, Sheriff of Calauag, Quezon ousted petitioner's representative from the land in question and put private respondents in possession thereof. 10

On March 10, 1971, petitioner filed a "Comment on Respondents' 'Motion for Reconsideration' dated January 29, 1971' and

'Supplemental Motion for Reconsideration and Manifestation,'" contending that the tender of deposit mentioned in the Suplemental

Motion was not really and officially made, "inasmuch as the same is notsupported by any official receipt from the lower court, or

from its clerk or cashier, as required by law;" that said deposit does not constitute sufficient compliance with the judgment sought

to be enforced, neither was it legally and validly made because the requisites for consignation had not been complied with; that the tender of legal interest for six months cannot substitute petitioner's enjoyment of the fruits of the property as long as the judgment

in Civil Case No. C-90 has not been implemented in the manner decreed therein; that contrary to the allegations of private

respondents, the value of the improvements on the whole property had been determined by the lower court, and the segregation of

the improvements for each lot should have been raised by them at the opportune moment by asking for the modification of the

decision before it became final and executory; and that the tolls on the property constituted "civil fruits" to which the petitioner is

entitled under the terms of the decision.

IV

The issue decisive of the controvery is—after the rendition by the trial court of its judgment in Civil Case No. C-90 on March 22,

1966 confirming the award of one-half of the property to Quirino Comintan—whether or not petitioner is still entitled to retain for

his own exclusive benefit all the fruits of the property, such as the tolls collected by him from March 1967 to December 1968, and

September 1969 to March 31, 1970, amounting to about P25,000.00. In other words, petitioner contends that so long as the

aforesaid amount of P13,632,00 decreed in the judgment representing the expenses for clearing the land and the value of the

coconuts and fruit trees planted by him remains unpaid, he can appropriate for his exclusive benefit all the fruits which he may derive from the property, without any obligation to apply any portion thereof to the payment of the interest and the principal of the

debt.

We find this contention untenable.

There is no question that a possessor in good faith is entitled to the fruits received before the possession is legally

interrupted. 11 Possession in good faith ceases or is legally interrupted from the moment defects in the title are made known to the

22

possessor, by extraneous evidence or by the filing of an action in court by the true owner for the recovery of the property. 12 Hence,

all the fruits that the possessor may receive from the time he is summoned in court, or when he answers the complaint, must be

delivered and paid by him to the owner or lawful possessor. 13

However, even after his good faith ceases, the possessor in fact can still retain the property, pursuant to Article 546 of the New

Civil Code, until he has been fully reimbursed for all the necessary and useful expenses made by him on the property. This right of

retention has been considered as one of the conglomerate of measures devised by the law for the protection of the possessor in

good faith. Its object is to guarantee the reimbursement of the expenses, such as those for the preservation of the property, 14 or

for the enhancement of its utility or productivity.15 It permits the actual possessor to remain in possession while he has not been

reimbursed by the person who defeated him in the possession for those necessary expenses and useful improvements made by him

on the thing possessed. The principal characteristic of the right of retention is its accessory character. It is accessory to a principal

obligation. Considering that the right of the possessor to receive the fruits terminates when his good faith ceases, it is necessary, in

order that this right to retain may be useful, to concede to the creditor the right to secure reimbursement from the fruits of the property by utilizing its proceeds for the payment of the interest as well as the principal of the debt while he remains in possession.

This right of retention of the property by the creditor, according to Scaevola, in the light of the provisions of Article 502 of the

Spanish Civil Code, 16 is considered not a coercive measure to oblige the debtor to pay, depriving him temporarily of the enjoyment

of the fruits of his property, but as a means of obtainitig compensation for the debt. The right of retention in this case is analogous

to a contract of antichresis and it cati be considered as a means of extinguishing the obligation, inasmuch as the right to retain the

thing lasts only for the period necessary to enable the creditor to be reimbursed from the fruits for the necessary and useful

expenses. 17

According to Manresa, the right of retention is, therefore, analogous to that of a pledge, if the property retained is a movable, and

to that of antichresis, if the property held is immovable. 18 This construction appears to be in harmony with similar provisions of the

civil law which employs the right of retention as a means or device by which a creditor is able to obtain the payment of a debt.

Thus, under Article 1731 of the New Civil Code, any person who has performed work upon a movable has a right to retain it by way

of pledge until he is paid. Similarly, under Article 1914 of the same Code, the agent may retain in pledge the things which are the

object of the agency until the principal effects reimbursement of the funds advanced by the former for the execution of the agency,

or he is indemnified for all damages which he may have suffered as a consequence of the execution of the agency, provided he is

free from fault. To the same effect, the depositary, under Article 1994 of the same Code, may retain the thing in pledge until the full payment of what may be due him by reason of the deposit. The usufructuary, pursuant to Article 612 of the same Code, may

retain the property until he is reimbursed for the amount paid for taxes levied on the capital (Article 597) and tor extraordinary

repairs (Article 594).

In all of these cases, the right of retention is used as a means of extinguishing the obligation. As amply observed by Manresa: "El

derecho de retencion, lo hemos dicho, es el derecho de prenda o el de anticresis constituido por la ley con independencia de las

partes." 19 In a pledge, if the thing pledged earns or produces fruits, income, dividends or interests, the creditor shall compensate

what he receives with those which are owing him. 20 In the same manner, in a contract of antichresis, the creditor acquires the right to receive the fruits of an immovable of his debtor with the obligation to apply them to payment of the interest, if owing, and

thereafter to the principal of his credit.21 The debtor can not reacquire enjoyment of the immovable until he has actually paid what

he owes the creditor. 22

Applying the afore-cited principles to the case at bar, petitioner cannot appropriate for his own exclusive benefit the tolls which he

collected from the property retained by him. It was his duty under the law, after deducting the necessary expenses for his

administration, to apply such amount collected to the payment of the interest, and the balance to the payment of the obligation.

We hold, therefore, that the disputed tolls, after deducting petitioner's expenses for administration, belong to Quirino Comintan,

owner of the land through which the toll road passed, further considering that the same was on portions of the property on which

petitioner had not introduced any improvement. The trial court itself clarified this matter when it placed the toll road under

receivership. The omission of any mention of the tolls in the decision itself may be attributed to the fact that the tolls appear to

have been collected after the rendition of the judgment of the trial court.

The records further reveal that earnest efforts have been made by private respondents to have the judgment executed in the most

practicable manner. They deposited in court the amount of the judgment in the sum of P13,632.00 in cash, subject only to the

accounting of the tolls collected by the petitioner so that whatever is due from him may be set off with the amount of

reimbursement. This is just and proper under the circumstances and, under the law, compensation or set off may take place, either

totally or partially. Considering that petitioner is the creditor with respect to the judgment obligation and the debtor with respect to

the tolls collected, Comintan being the owner thereof, the trial court's order for an accounting and compensation is in accord with

law. 23

With respect to the amount of reimbursement to be paid by Comintan, it appears that the dispositive portion of the decision was

lacking in specificity, as it merely provided that Comintan and Zamora are jointly liable therefor. When two persons are liable under

a contract or under a judgment, and no words appear in the contract or judgment to make each liable for the entire obligation, the

presumption is that their obligation is joint ormancomunada, and each debtor is liable only for a proportionate part of the

obligation. 24 The judgment debt of P13,632.00 should, therefore, be pro-rated in equal shares to Comintan and Zamora.

Regarding Lot 5785-B, it appears that no public sale has yet been conducted by the Bureau of Lands and, therefore, petitioner is

entitled to remain in possession thereof. This is not disputed by respondent Eleuterio Zamora. 25 After public sale is had and in the

event that Ortiz is not declared the successful bidder, then he should be reimbursed by respondent Zamora in the corresponding

amount for the improvements on Lot 5785-B.

23

WHEREFORE, in view hereof, the Order of respondent Court of November 18, 1970 is hereby modified to conform to the foregoing

judgment. The Writ of Preliminary Injunction, dated January 29, 1971, is hereby dissolved. Without special pronouncement as to

costs.

Barredo (Chairman), Concepcion, Jr. and Guerrero, JJ., concur.1äwphï1.ñët

Aquino, J., concurs in the result.

Santos and Abad Santos, JJ., are on leave.

Guerrero, J., was designated to sit in the Second Division

FEDERICO GEMINIANO, MARIA GEMINIANO, ERNESTO GEMINIANO, ASUNCION GEMINIANO, LARRY GEMINIANO, and

MARLYN GEMINIANO,petitioners, vs. COURT OF APPEALS, DOMINADOR NICOLAS, and MARY A.

NICOLAS, respondents.

D E C I S I O N

DAVIDE, JR., J.:

This petition for review on certiorari has its origins in Civil Case No. 9214 of Branch 3 of the Municipal Trial Court in Cities

(MTCC) in Dagupan City for unlawful detainer and damages. The petitioners ask the Court to set aside the decision of the Court of Appeals affirming the decision of Branch 40 of the Regional Trial Court (RTC) of Dagupan City, which, in turn, reversed the MTCC;

ordered the petitioners to reimburse the private respondents the value of the house in question and other improvements; and

allowed the latter to retain the premises until reimbursement was made.

It appears that Lot No. 3765-B-1 containing an area of 314 square meters was originally owned by the petitioners' mother,

Paulina Amado vda. de Geminiano. On a 12-square-meter portion of that lot stood the petitioners' unfinished bungalow, which the

petitioners sold in November 1978 to the private respondents for the sum of P6,000.00, with an alleged promise to sell to the latter

that portion of the lot occupied by the house. Subsequently, the petitioners' mother executed a contract of lease over a 126

square-meter portion of the lot, including that portion on which the house stood, in favor of the private respondents for P40.00 per

month for a period of seven years commencing on 15 November 1978.[1] The private respondents then introduced additional improvements and registered the house in their names. After the expiration of the lease contract in November 1985, however, the

petitioners' mother refused to accept the monthly rentals.

It turned out that the lot in question was the subject of a suit, which resulted in its acquisition by one Maria Lee in 1972. In

1982, Lee sold the lot to Lily Salcedo, who in turn sold it in 1984 to the spouses Agustin and Ester Dionisio.

On 14 February 1992, the Dionisio spouses executed a Deed of Quitclaim over the said property in favor of the

petitioners.[2] As such, the lot was registered in the latter's names.[3]

On 9 February 1993, the petitioners sent, via registered mail, a letter addressed to private respondent Mary Nicolas

demanding that she vacate the premises and pay the rentals in arrears within twenty days from notice.[4]

Upon failure of the private respondents to heed the demand, the petitioners filed with the MTCC of Dagupan City a complaint

for unlawful detainer and damages.

During the pre-trial conference, the parties agreed to confine the issues to: (1) whether there was an implied renewal of the

lease which expired in November 1985; (2) whether the lessees were builders in good faith and entitled to reimbursement of the

value of the house and improvements; and (3) the value of the house.

The parties then submitted their respective position papers and the case was heard under the Rule on Summary Procedure.

On the first issue, the court held that since the petitioners' mother was no longer the owner of the lot in question at the t ime

the lease contract was executed in 1978, in view of its acquisition by Maria Lee as early as 1972, there was no lease to speak of,

much less, a renewal thereof. And even if the lease legally existed, its implied renewal was not for the period stipulated in the

original contract, but only on a month-to-month basis pursuant to Article 1687 of the Civil Code. The refusal of the petitioners'

mother to accept the rentals starting January 1986 was then a clear indication of her desire to terminate the monthly lease. As regards the petitioners' alleged failed promise to sell to the private respondents the lot occupied by the house, the court held that

such should be litigated in a proper case before the proper forum, not an ejectment case where the only issue was physical

possession of the property.

The court resolved the second issue in the negative, holding that Articles 448 and 546 of the Civil Code, which allow

possessors in good faith to recover the value of improvements and retain the premises until reimbursed, did not apply to lessees

like the private respondents, because the latter knew that their occupation of the premises would continue only during the life of

the lease. Besides, the rights of the private respondents were specifically governed by Article 1678, which allows reimbursement of

up to one-half of the value of the useful improvements, or removal of the improvements should the lessor refuse to reimburse.

On the third issue, the court deemed as conclusive the private respondents' allegation that the value of the house and

improvements was P180,000.00, there being no controverting evidence presented.

24

The trial court thus ordered the private respondents to vacate the premises, pay the petitioners P40.00 a month as

reasonable compensation for their stay thereon from the filing of the complaint on 14 April 1993 until they vacated, and to pay the

sum of P1,000.00 as attorney's fees, plus costs.[5]

On appeal by the private respondents, the RTC of Dagupan City reversed the trial court's decision and rendered a new

judgment: (1) ordering the petitioners to reimburse the private respondents for the value of the house and improvements in the amount of P180,000.00 and to pay the latter P10,000.00 as attorney's fees and P2,000.00 as litigation expenses; and (2) allowing

the private respondents to remain in possession of the premises until they were fully reimbursed for the value of the house.[6] It

ruled that since the private respondents were assured by the petitioners that the lot they leased would eventually be sold to them,

they could be considered builders in good faith, and as such, were entitled to reimbursement of the value of the house and

improvements with the right of retention until reimbursement had been made.

On appeal, this time by the petitioners, the Court of Appeals affirmed the decision of the RTC[7] and denied[8] the petitioners'

motion for reconsideration. Hence, the present petition.

The Court is confronted with the issue of which provision of law governs the case at bench: Article 448 or Article 1678 of the

Civil Code? The said articles read as follows:

Art. 448. The owner of the land on which anything has been built, sown or planted in good faith, shall have the right to appropriate as his own the works, sowing or planting, after payment of the indemnity provided for in articles 546 and 548, or to oblige the one

who built or planted to pay the price of the land, and the one who sowed, the proper rent. However, the builder or planter cannot

be obliged to buy the land if its value is considerably more than that of the building or trees. In such case, he shall pay reasonable

rent, if the owner of the land does not choose to appropriate the building or trees after proper indemnity. The parties shall agree

upon the terms of the lease and in case of disagreement, the court shall fix the terms thereof.

xxx xxx xxx

Art. 1678. If the lessee makes, in good faith, useful improvements which are suitable to the use for which the lease is intended,

without altering the form or substance of the property leased, the lessor upon the termination of the lease shall pay the lessee one-

half of the value of the improvements at that time. Should the lessor refuse to reimburse said amount, the lessee may remove the improvements, even though the principal thing may suffer damage thereby. He shall not, however, cause any more impairment

upon the property leased than is necessary.

With regard to ornamental expenses, the lessee shall not be entitled to any reimbursement, but he may remove the ornamental

objects, provided no damage is caused to the principal thing, and the lessor does not choose to retain them by paying their value at

the time the lease is extinguished.

The crux of the said issue then is whether the private respondents are builders in good faith or mere lessees.

The private respondents claim they are builders in good faith, hence, Article 448 of the Civil Code should apply. They rely on

the lack of title of the petitioners' mother at the time of the execution of the contract of lease, as well as the alleged assurance

made by the petitioners that the lot on which the house stood would be sold to them.

It has been said that while the right to let property is an incident of title and possession, a person may be a lessor and occupy

the position of a landlord to the tenant although he is not the owner of the premises let.[9] After all, ownership of the property is not

being transferred,[10] only the temporary use and enjoyment thereof.[11]

In this case, both parties admit that the land in question was originally owned by the petitioners' mother. The land was allegedly acquired later by one Maria Lee by virtue of an extrajudicial foreclosure of mortgage. Lee, however, never sought a writ

of possession in order that she gain possession of the property in question.[12] The petitioners' mother therefore remained in

possession of the lot.

It is undisputed that the private respondents came into possession of a 126 square-meter portion of the said lot by virtue of a

contract of lease executed by the petitioners' mother in their favor. The juridical relation between the petitioners' mother as lessor,

and the private respondents as lessees, is therefore well-established, and carries with it a recognition of the lessor's title.[13] The

private respondents, as lessees who had undisturbed possession for the entire term under the lease, are then estopped to deny

their landlord's title, or to assert a better title not only in themselves, but also in some third person while they remain in possession

of the leased premises and until they surrender possession to the landlord.[14] This estoppel applies even though the lessor had no title at the time the relation of lessor and lessee was created,[15]and may be asserted not only by the original lessor, but also by

those who succeed to his title.[16]

Being mere lessees, the private respondents knew that their occupation of the premises would continue only for the life of the

lease. Plainly, they cannot be considered as possessors nor builders in good faith.[17]

In a plethora of cases,[18] this Court has held that Article 448 of the Civil Code, in relation to Article 546 of the same Code,

which allows full reimbursement of useful improvements and retention of the premises until reimbursement is made, applies only to

a possessor in good faith,i.e., one who builds on land with the belief that he is the owner thereof. It does not apply where one's

only interest is that of a lessee under a rental contract; otherwise, it would always be in the power of the tenant to "improve" his

landlord out of his property.

Anent the alleged promise of the petitioners to sell the lot occupied by the private respondents' house, the same was not substantiated by convincing evidence. Neither the deed of sale over the house nor the contract of lease contained an option in

favor of the respondent spouses to purchase the said lot. And even if the petitioners indeed promised to sell, it would not make the

private respondents possessors or builders in good faith so as to be covered by the provisions of Article 448 of the Civil Code. The

25

latter cannot raise the mere expectancy of ownership of the aforementioned lot because the alleged promise to sell was not fulfilled

nor its existence even proven. The first thing that the private respondents should have done was to reduce the alleged promise

into writing, because under Article 1403 of the Civil Code, an agreement for the sale of real property or an interest therein is

unenforceable, unless some note or memorandum thereof be produced. Not having taken any steps in order that the alleged

promise to sell may be enforced, the private respondents cannot bank on that promise and profess any claim nor color of title over the lot in question.

There is no need to apply by analogy the provisions of Article 448 on indemnity as was done in Pecson vs. Court of

Appeals,[19] because the situation sought to be avoided and which would justify the application of that provision, is not present in

this case. Suffice it to say, "a state of forced co-ownership" would not be created between the petitioners and the private

respondents. For, as correctly pointed out by the petitioners, the rights of the private respondents as lessees are governed by

Article 1678 of the Civil Code which allows reimbursement to the extent of one-half of the value of the useful improvements.

It must be stressed, however, that the right to indemnity under Article 1678 of the Civil Code arises only if the lessor opts to

appropriate the improvements. Since the petitioners refused to exercise that option,[20] the private respondents cannot compel

them to reimburse the one-half value of the house and improvements. Neither can they retain the premises until reimbursement is

made. The private respondents' sole right then is to remove the improvements without causing any more impairment upon the property leased than is necessary.[21]

WHEREFORE, judgment is hereby rendered GRANTING the instant petition; REVERSING and SETTING ASIDE the decision of

the Court of Appeals of 27 January 1995 in CA-G.R. SP No. 34337; and REINSTATING the decision of Branch 3 of the Municipal Trial

Court in Cities of Dagupan City in Civil Case No. 9214 entitled "Federico Geminiano, et al. vs. Dominador Nicolas, et al."

Costs against the private respondents.

SO ORDERED.

PLEASANTVILLE DEVELOPMENT CORPORATION, petitioner, vs. COURT OF APPEALS, WILSON KEE, C.T. TORRES

ENTERPRISES, INC. and ELDRED JARDINICO, respondents.

D E C I S I O N

PANGANIBAN, J.:

Is a lot buyer who constructs improvements on the wrong property erroneously delivered by the owner’s agent, a builder in

good faith? This is the main issue resolved in this petition for review on certiorari to reverse the Decision[1] of the Court of

Appeals[2] in CA-G.R. SP No. 11040, promulgated on August 20, 1987.

By resolution dated November 13, 1995, the First Division of this Court resolved to transfer this case (along with several

others) to the Third Division. After due deliberation and consultation, the Court assigned the writing of this Decision to the undersigned ponente.

The Facts

The facts, as found by respondent Court, are as follows:

Edith Robillo purchased from petitioner a parcel of land designated as Lot 9, Phase II and located at Taculing Road,

Pleasantville Subdivision, Bacolod City. In 1975, respondent Eldred Jardinico bought the rights to the lot from Robillo. At that

time, Lot 9 was vacant.

Upon completing all payments, Jardinico secured from the Register of Deeds of Bacolod City on December 19, 1978 Transfer

Certificate of Title No. 106367 in his name. It was then that he discovered that improvements had been introduced on Lot 9 by

respondent Wilson Kee, who had taken possession thereof.

It appears that on March 26, 1974, Kee bought on installment Lot 8 of the same subdivision from C.T. Torres Enterprises,

Inc. (CTTEI), the exclusive real estate agent of petitioner. Under the Contract to Sell on Installment, Kee could possess the lot even

before the completion of all installment payments. On January 20, 1975, Kee paid CTTEI the relocation fee of P50.00 and another

P50.00 on January 27, 1975, for the preparation of the lot plan. These amounts were paid prior to Kee’s taking actual possession

of Lot 8. After the preparation of the lot plan and a copy thereof given to Kee, CTTEI through its employee, Zenaida Octaviano,

accompanied Kee’s wife, Donabelle Kee, to inspect Lot 8. Unfortunately, the parcel of land pointed by Octaviano

wasLot 9. Thereafter, Kee proceeded to construct his residence, a store, an auto repair shop and other improvements on the lot.

After discovering that Lot 9 was occupied by Kee, Jardinico confronted him. The parties tried to reach an amicable

settlement, but failed.

On January 30, 1981, Jardinico’s lawyer wrote Kee, demanding that the latter remove all improvements and vacate Lot 9. When Kee refused to vacate Lot 9, Jardinico filed with the Municipal Trial Court in Cities, Branch 3, Bacolod City

(MTCC), a complaint for ejectment with damages against Kee.

Kee, in turn, filed a third-party complaint against petitioner and CTTEI.

26

The MTCC held that the erroneous delivery of Lot 9 to Kee was attributable to CTTEI. It further ruled that petitioner and

CTTEI could not successfully invoke as a defense the failure of Kee to give notice of his intention to begin construction required

under paragraph 22 of the Contract to Sell on Installment and his having built a sari-sari store without. the prior approval of

petitioner required under paragraph 26 of said contract, saying that the purpose of these requirements was merely to regulate the

type of improvements to be constructed on the lot[3].

However, the MTCC found that petitioner had already rescinded its contract with Kee over Lot8 for the latter’s failure to pay

the installments due, and that Kee had not contested the rescission. The rescission was effected in 1979, before the complaint was

instituted. The MTCC concluded that Kee no longer had any right over the lot subject of the contract between him and

petitioner. Consequently, Kee must pay reasonable rentals for the use of Lot 9, and, furthermore, he cannot claim reimbursement

for the improvements he introduced on said lot.

The MTCC thus disposed:

“IN VIEW OF ALL THE FOREGOING, judgment is hereby rendered as follows:

1. Defendant Wilson Kee is ordered to vacate tithe premises of Lot 9, covered by TCT No. 106367 and to remove all structures and

improvements he introduced thereon;

2. Defendant Wilson Kee is ordered to pay to the plaintiff rentals at the rate of P 15.00 a day computed from the time this suit was filed on March 12, 1981 until he actually vacates the premises. This amount shall bear interests (sic) at the rate of 12 per cent

(sic) per annum.

3. Third-Party Defendant CT. Torres Enterprises, Inc. and Pleasantville Subdivision are ordered to pay the plaintiff jointly and

severally the sum of P3,000.00 as attorney’s fees and P700.00 as cost and litigation expenses.”[4]

On appeal, the Regional Trial Court, Branch 48, Bacolod City (RTC) ruled that petitioner and CTTEI were not at fault or were

not negligent, there being no preponderant evidence to show that they directly participated in the delivery of Lot 9 to Kee.[5] It

found Kee a builder in bad faith. It further ruled that even assuming arguendo that Kee was acting in good faith, he was,

nonetheless, guilty of unlawfully usurping the possessory right of Jardinico over Lot 9 from the time he was served with notice to

vacate said lot, and thus was liable for rental.

The RTC thus disposed:

“WHEREFORE, the decision appealed from is affirmed with respect to the order against the defendant to vacate the premises of Lot No. 9 covered by Transfer Certificate of Title No. T-106367 of the land records of Bacolod City; the removal of all structures and

improvements introduced thereon at his expense and the payment to plaintiff (sic) the sum of Fifteen (P 15.00) Pesos a day as

reasonable rental to be computed from January 30, 1981, the date of the demand, and not from the date of the filing of the

complaint, until he had vacated (sic) the premises, with interest thereon at 12% per annum. This Court further renders judgment

against the defendant to pay the plaintiff the sum of Three Thousand (P3,000.00) Pesos as attorney’s fees, plus costs of litigation.

“The third-party complaint against Third-Party Defendants Pleasantville Development Corporation and C.T. Torres Enterprises, Inc.

is dismissed. The order against Third-Party Defendants to pay attorney’s fees to plaintiff and costs of litigation is reversed.”[6]

Following the denial of his motion for reconsideration on October 20, 1986, Kee appealed directly to the Supreme Court,

which referred the matter to the Court of Appeals.

The appellate court ruled that Kee was a builder in good faith, as he was unaware of the “mix-up” when he began

construction of the improvements on Lot 8. It further ruled that the erroneous delivery was due to the negligence of CTTEI, and that such wrong delivery was likewise imputable to its principal, petitioner herein. The appellate court also ruled that the award of

rentals was without basis.

Thus, the Court of Appeals disposed:

“WHEREFORE, the petition is GRANTED, the appealed decision is REVERSED, and judgment is rendered as follows:

1. Wilson Kee is declared a builder in good faith with respect to the improvements he introduced on Lot 9, and is

entitled to the rights granted him under Articles 448, 546 and 548 of the New Civil Code.

2. Third-party defendants C.T. Torres Enterprises, Inc. and Pleasantville Development Corporation are solidarily liable

under the following circumstances:

a. If Eldred Jardinico decides to appropriate the improvements and, thereafter, remove these structures, the third-party defendants shall answer for all demolition expenses and the value of the improvements thus destroyed or

rendered useless;

b. If Jardinico prefers that Kee buy the land, the third-party defendants shall answer for the amount representing the

value of Lot 9 that Kee should pay to Jardinico.

27

3. Third-party defendants C.T. Torres Enterprises, Inc. and Pleasantville Development Corporation are ordered to

pay in solidum the amount of P3,000.00 to Jardinico as attorney’s fees, as well as litigation expenses.

4. The award of rentals to Jardinico is dispensed with.

“Furthermore, the case is REMANDED to the court of origin for the determination of the actual value of the improvements and the property (Lot 9), as well as for further proceedings in conformity with Article 448 of the New Civil Code.”[7]

Petitioner then filed the instant petition against Kee, Jardinico and CTTEI.

The Issues

The petition submitted the following grounds to justify a review of the respondent Court’s Decision, as follows:

“1. The Court of Appeals has decided the case in a way probably not in accord with law or the the (sic) applicable decisions of the

Supreme Court on third-party complaints, by ordering third-party defendants to pay the demolition expenses and/or price of the

land;

“2. The Court of Appeals has so far departed from the accepted course of judicial proceedings, by granting to private respondent-Kee the rights of a builder in good faith in excess of what the law provides, thus enriching private respondent Kee at the expense of

the petitioner;

“3. In the light of the subsequent events or circumstances which changed the rights of the parties, it becomes imperative to set

aside or at least modify the judgment of the Court of Appeals to harmonize with justice and the facts;

“4. Private respondent-Kee in accordance with the findings of facts of the lower court is clearly a builder in bad faith, having

violated several provisions of the contract to sell on installments;

“5. The decision of the Court of Appeals, holding the principal, Pleasantville Development Corporation (liable) for the acts made by

the agent in excess of its authority is clearly in violation of the provision of the law;

“6. The award of attorney’s fees is clearly without basis and is equivalent to putting a premium in (sic) court litigation.”

From these grounds, the issues could be re-stated as follows:

(1) Was Kee a builder in good faith?

(2) What is the liability, if any, of petitioner and its agent, C.T. Torres Enterprises, Inc.? and

(3) Is the award of attorney’s fees proper?

The First Issue: Good Faith

Petitioner contends that the Court of Appeals erred in reversing the RTC’s ruling that Kee was a builder in bad faith.

Petitioner fails to persuade this Court to abandon the findings and conclusions of the Court of Appeals that Kee was a builder

in good faith. We agree with the following observation of the Court of Appeals:

“The roots of the controversy can be traced directly to the errors committed by CTTEI, when it pointed the wrong property to Wilson Kee and his wife. It is highly improbable that a purchaser of a lot would knowingly and willingly build his residence on a lot

owned by another, deliberately exposing himself and his family to the risk of being ejected from the land and losing all

improvements thereon, not to mention the social humiliation that would follow.

“Under the circumstances, Kee had acted in the manner of a prudent man in ascertaining the identity of his property. Lot 8 is

covered by Transfer Certificate of Title No. T-69561, while Lot 9 is identified in Transfer Certificate of Title No. T-106367. Hence,

under the Torrens system of land registration, Kee is presumed to have knowledge of the metes and bounds of the property with

which he is dealing. x x x

28

xxx xxx xxx

“But as Kee is a layman not versed in the technical description of his property, he had to find a way to ascertain that what was described in TCT No. 69561 matched Lot 8. Thus, he went to the subdivision developer’s agent and applied and paid for the

relocation of the lot, as well as for the production of a lot plan by CTTEI’s geodetic engineer. Upon Kee’s receipt of the map, his wife

went to the subdivision site accompanied by CTTEI’s employee, Octaviano, who authoritatively declared that the land she was

pointing to was indeed Lot 8. Having full faith and confidence in the reputation of CTTEI, and because of the company’s positive

identification of the property, Kee saw no reason to suspect that there had been a misdelivery. The steps Kee had taken to protect

his interests were reasonable. There was no need for him to have acted ex-abundantia cautela, such as being present during the

geodetic engineer’s relocation survey or hiring an independent geodetic engineer to countercheck for errors, for the final delivery of

subdivision lots to their owners is part of the regular course of everyday business of CTTEI. Because of CTTEI’s blunder, what Kee

had hoped to forestall did in fact transpire. Kee’s efforts all went to naught.”[8]

Good faith consists in the belief of the builder that the land he is building on is his and his ignorance of any defect or flaw in

his title.[9] And as good faith is presumed, petitioner has the burden of proving bad faith on the part of Kee.[10]

At the time he built improvements on Lot 8, Kee believed that said lot was what he bought from petitioner. He was not

aware that the lot delivered to him was not Lot 8. Thus, Kee’s good faith. Petitioner failed to prove otherwise.

To demonstrate Kee’s bad faith, petitioner points to Kee’s violation of paragraphs 22 and 26 of the Contract of Sale on

Installment.

We disagree. Such violations have no bearing whatsoever on whether Kee was a builder in good faith, that is, on his state of mind at the time he built the improvements on Lot 9. These alleged violations may give rise to petitioner’s cause of action against

Kee under the said contract (contractual breach), but may not be bases to negate the presumption that Kee was a builder in good

faith.

Petitioner also points out that, as found by the trial court, the Contract of Sale on Installment covering Lot 8 between it and

Kee was rescinded long before the present action was instituted. This has no relevance on the liability of petitioner, as such fact

does not negate the negligence of its agent in pointing out the wrong lot to Kee. Such circumstance is relevant only as it gives

Jardinico a cause of action for unlawful detainer against Kee.

Petitioner next contends that Kee cannot “claim that another lot was erroneously pointed out to him” because the latter

agreed to the following provision in the Contract of Sale on Installment, to wit:

“13. The Vendee hereby declares that prior to the execution of his contract he/she has personally examined or inspected the

property made subject-matter hereof, as to its location, contours, as well as the natural condition of the lots and from the date

hereof whatever consequential change therein made due to erosion, the said Vendee shall bear the expenses of the necessary

fillings, when the same is so desired by him/her.”[11]

The subject matter of this provision of the contract is the change of the location, contour and condition of the lot due to

erosion. It merely provides that the vendee, having examined the property prior to the execution of the contract, agrees to

shoulder the expenses resulting from such change.

We do not agree with the interpretation of petitioner that Kee contracted away his right to recover damages resulting from

petitioner’s negligence. Such waiver would be contrary to public policy and cannot be allowed. “Rights may be waived, unless the

waiver is contrary to law, public order, public policy, morals, or good customs, or prejudicial to a third person with a right

recognized by law.”[12]

The Second Issue: Petitioner’s Liability

Kee filed a third-party complaint against petitioner and CTTEI, which was dismissed by the RTC after ruling that there was no evidence from which fault or negligence on the part of petitioner and CTTEI can be inferred. The Court of Appeals disagreed and

found CTTEI negligent for the erroneous delivery of the lot by Octaviano, its employee.

Petitioner does not dispute the fact that CTTEI was its agent. But it contends that the erroneous delivery of Lot 9 to Kee was

an act which was clearly outside the scope of its authority, and consequently, CTTEI alone should be liable. It asserts that “while

[CTTEI] was authorized to sell the lot belonging to the herein petitioner, it was never authorized to deliver the wrong lot to Kee.”[13]

Petitioner’s contention is without merit.

The rule is that the principal is responsible for the acts of the agent, done within the scope of his authority, and should bear

the damage caused to third persons.[14] On the other hand, the agent who exceeds his authority is personally liable for the

damage.[15]

CTTEI was acting within its authority as the sole real estate representative of petitioner when it made the delivery to Kee. In

acting within its scope of authority, it was, however, negligent. It is this negligence that is the basis of petitioner’s liability, as

principal of CTTEI, per Articles 1909 and 1910 of the Civil Code.

Pending resolution of the case before the Court of Appeals, Jardinico and Kee on July 24, 1987 entered into a deed of sale,

wherein the former sold Lot 9 to Kee. Jardinico and Kee did not inform the Court of Appeals of such deal.

29

The deed of sale contained the following provision:

“1. That Civil Case No. 3815 entitled “Jardinico vs. Kee” which is now pending appeal with the Court of Appeals, regardless of the outcome of the decision shall be mutually disregarded and shall not be pursued by the parties herein and shall be considered

dismissed and without effect whatsoever;[16]

Kee asserts though that the “terms and conditions in said deed of sale are strictly for the parties thereto” and that “(t)here is

no waiver made by either of the parties in said deed of whatever favorable judgment or award the honorable respondent Court of

Appeals may make in their favor against herein petitioner Pleasantville Development Corporation and/or private respondent C.T.

Torres Enterprises, Inc.”[17]

Obviously, the deed of sale can have no effect on the liability of petitioner. As we have earlier stated, petitioner’s liability is

grounded on the negligence of its agent. On the other hand, what the deed of sale regulates are the reciprocal rights of Kee and

Jardinico; it stressed that they had reached an agreement independent of the outcome of the case.

Petitioner further assails the following holding of the Court of Appeals:

“2. Third-party defendants C.T. Torres Enterprises, Inc. and Pleasantville Development Corporation are solidarily liable under the

following circumstances:

“a. If Eldred Jardinico decides to appropriate the improvements and, thereafter, remove these structures, the third-

party defendants shall answer for all demolition expenses and the value of the improvements thus destroyed or

rendered useless;

“b. If Jardinico prefers that Kee buy the land, the third-party defendants shall answer for the amount representing the value of Lot 9 that Kee should pay to Jardinico.”[18]

Petitioner contends that if the above holding would be carried out, Kee would be unjustly enriched at its expense. In other

words, Kee would be -able to own the lot, as buyer, without having to pay anything on it, because the aforequoted portion of

respondent Court’s Decision would require petitioner and CTTEI jointly and solidarily to “answer” or reimburse Kee there for.

We agree with petitioner.

Petitioner’s liability lies in the negligence of its agent CTTEI. For such negligence, the petitioner should be held liable for damages. Now, the extent and/or amount of damages to be awarded is a factual issue which should be determined after evidence

is adduced. However, there is no showing that such evidence was actually presented in the trial court; hence no damages could

now be awarded.

The rights of Kee and Jardinico vis-a-vis each other, as builder in good faith and owner in good faith, respectively, are

regulated by law (i.e., Arts. 448, 546 and 548 of the Civil Code). It was error for the Court of Appeals to make a “slight

modification” in the application of such law, on the ground of “equity”. At any rate, as it stands now, Kee and Jardinico have

amicably settled through their deed of sale their rights and obligations with regards to Lot 9. Thus, we delete items 2 (a) and (b) of

the dispositive portion of the Court of Appeals’ Decision [as reproduced above] holding petitioner and CTTEI solidarily liable.

The Third Issue: Attorney’s Fees

The MTCC awarded Jardinico attorney’s fees and costs in the amount of P3,000.00 and P700.00, respectively, as prayed for in his complaint. The RTC deleted the award, consistent with its ruling that petitioner was without fault or negligence. The Court of

Appeals, however, reinstated the award of attorney’s fees after ruling that petitioner was liable for its agent’s negligence.

The award of attorney’s fees lies within the discretion of the court and depends upon the circumstances of each case.[19] We

shall not interfere with the discretion of the Court of Appeals. Jardinico was compelled to litigate for the protection of his interests

and for the recovery of damages sustained as a result of the negligence of petitioner’s agent.[20]

In sum, we rule that Kee is a builder in good faith. The disposition of the Court of Appeals that Kee “is entitled to the rights

granted him under Articles 448, 546 and 548 of the New Civil Code” is deleted, in view of the deed of sale entered into by Kee and

Jardinico, which deed now governs the rights of Jardinico and Kee as to each other. There is also no further need, as ruled by the

appellate Court, to remand the case to the court of origin “for determination of the actual value of the improvements and the property (Lot 9), as well as for further proceedings in conformity with Article 448 of the New Civil Code.”

WHEREFORE, the petition is partially GRANTED. The Decision of the Court of Appeals is hereby MODIFIED as follows:

(1) Wilson Kee is declared a builder in good faith;

(2) Petitioner Pleasantville Development Corporation and respondent C.T. Tones Enterprises, Inc. are declared

solidarily liable for damages due to negligence; however, since the amount and/or extent of such damages was

not proven during the trial, the same cannot now be quantified and awarded;

(3) Petitioner Pleasantville Develpment Corporation and respondent C.T. Torres Enterprises, Inc. are ordered to pay

in solidum the amount of P3,000.00 to Jardinico as attorney’s fees, as well as litigation expenses; and

30

(4) The award of rentals to Jardinico is dispensed with.

SO ORDERED.

SPOUSES JUAN NUGUID AND ERLINDA T. NUGUID, petitioners, vs. HON. COURT OF APPEALS AND PEDRO P.

PECSON, respondents.

D E C I S I O N

QUISUMBING, J.:

This is a petition for review on certiorari of the Decision[1] dated May 21, 2001, of the Court of Appeals in CA-G.R. CV No.

64295, which modified the Order dated July 31, 1998 of the Regional Trial Court (RTC) of Quezon City, Branch 101 in Civil Case No.

Q-41470. The trial court ordered the defendants, among them petitioner herein Juan Nuguid, to pay respondent herein Pedro P.

Pecson, the sum of P1,344,000 as reimbursement of unrealized income for the period beginning November 22, 1993 to December

1997. The appellate court, however, reduced the trial court’s award in favor of Pecson from the said P1,344,000 to P280,000. Equally assailed by the petitioners is the appellate court’s Resolution[2] dated January 10, 2002, denying the motion

for reconsideration.

It may be recalled that relatedly in our Decision dated May 26, 1995, in G.R. No. 115814, entitled Pecson v. Court of Appeals,

we set aside the decision of the Court of Appeals in CA-G.R. SP No. 32679 and the Order dated November 15, 1993, of the RTC of

Quezon City, Branch 101 and remanded the case to the trial court for the determination of the current market value of the four-

door two-storey apartment building on the 256-square meter commercial lot.

The antecedent facts in this case are as follows:

Pedro P. Pecson owned a commercial lot located at 27 Kamias Road, Quezon City, on which he built a four-door two-storey

apartment building. For failure to pay realty taxes, the lot was sold at public auction by the City Treasurer of Quezon City to Mamerto Nepomuceno, who in turn sold it for P103,000 to the spouses Juan and Erlinda Nuguid.

Pecson challenged the validity of the auction sale before the RTC of Quezon City in Civil Case No. Q-41470. In its

Decision,[3] dated February 8, 1989, the RTC upheld the spouses’ title but declared that the four-door two-storey apartment building

was not included in the auction sale.[4]This was affirmed in toto by the Court of Appeals and thereafter by this Court, in its

Decision[5]dated May 25, 1993, in G.R. No. 105360 entitled Pecson v. Court of Appeals.

On June 23, 1993, by virtue of the Entry of Judgment of the aforesaid decision in G.R. No. 105360, the Nuguids became the

uncontested owners of the 256-square meter commercial lot.

As a result, the Nuguid spouses moved for delivery of possession of the lot and the apartment building.

In its Order[6] of November 15, 1993, the trial court, relying upon Article 546[7] of the Civil Code, ruled that the Spouses

Nuguid were to reimburse Pecson for his construction cost ofP53,000, following which, the spouses Nuguid were entitled to immediate issuance of a writ of possession over the lot and improvements. In the same order the RTC also directed Pecson to pay

the same amount of monthly rentals to the Nuguids as paid by the tenants occupying the apartment units or P21,000 per month

from June 23, 1993, and allowed the offset of the amount of P53,000 due from the Nuguids against the amount of rents collected

by Pecson from June 23, 1993 to September 23, 1993 from the tenants of the apartment.[8]

Pecson duly moved for reconsideration, but on November 8, 1993, the RTC issued a Writ of Possession,[9] directing the deputy

sheriff to put the spouses Nuguid in possession of the subject property with all the improvements thereon and to eject all the

occupants therein.

Aggrieved, Pecson then filed a special civil action for certiorari and prohibition docketed as CA-G.R. SP No. 32679 with the

Court of Appeals.

In its decision of June 7, 1994, the appellate court, relying upon Article 448[10] of the Civil Code, affirmed the order of

payment of construction costs but rendered the issue of possession moot on appeal, thus:

WHEREFORE, while it appears that private respondents [spouses Nuguid] have not yet indemnified petitioner [Pecson] with the cost

of the improvements, since Annex I shows that the Deputy Sheriff has enforced the Writ of Possession and the premises have been

turned over to the possession of private respondents, the quest of petitioner that he be restored in possession of the premises is

rendered moot and academic, although it is but fair and just that private respondents pay petitioner the construction cost of P53,000.00; and that petitioner be ordered to account for any and all fruits of the improvements received by him starting on June

23, 1993, with the amount of P53,000.00 to be offset therefrom.

IT IS SO ORDERED.[11] [Underscoring supplied.]

Frustrated by this turn of events, Pecson filed a petition for review docketed as G.R. No. 115814 before this Court.

On May 26, 1995, the Court handed down the decision in G.R. No 115814, to wit:

WHEREFORE, the decision of the Court of Appeals in CA-G.R. SP No. 32679 and the Order of 15 November 1993 of the Regional

Trial Court, Branch 101, Quezon City in Civil Case No. Q-41470 are hereby SET ASIDE.

31

The case is hereby remanded to the trial court for it to determine the current market value of the apartment building on the

lot. For this purpose, the parties shall be allowed to adduce evidence on the current market value of the apartment building. The

value so determined shall be forthwith paid by the private respondents [Spouses Juan and Erlinda Nuguid] to the petitioner [Pedro

Pecson] otherwise the petitioner shall be restored to the possession of the apartment building until payment of the required

indemnity.

No costs.

SO ORDERED.[12] [Emphasis supplied.]

In so ruling, this Court pointed out that: (1) Article 448 of the Civil Code is not apposite to the case at bar where the owner of

the land is the builder, sower, or planter who then later lost ownership of the land by sale, but may, however, be applied by

analogy; (2) the current market value of the improvements should be made as the basis of reimbursement; (3) Pecson was entitled

to retain ownership of the building and, necessarily, the income therefrom; (4) the Court of Appeals erred not only in upholding the

trial court’s determination of the indemnity, but also in ordering Pecson to account for the rentals of the apartment building from

June 23, 1993 to September 23, 1993.

On the basis of this Court’s decision in G.R. No. 115814, Pecson filed a Motion to Restore Possession and a Motion to Render

Accounting, praying respectively for restoration of his possession over the subject 256-square meter commercial lot and for the spouses Nuguid to be directed to render an accounting under oath, of the income derived from the subject four-door apartment

from November 22, 1993 until possession of the same was restored to him.

In an Order[13] dated January 26, 1996, the RTC denied the Motion to Restore Possession to the plaintiff averring that the

current market value of the building should first be determined. Pending the said determination, the resolution of the Motion for

Accounting was likewise held in abeyance.

With the submission of the parties’ assessment and the reports of the subject realty, and the reports of the Quezon City

Assessor, as well as the members of the duly constituted assessment committee, the trial court issued the following

Order[14] dated October 7, 1997, to wit:

On November 21, 1996, the parties manifested that they have arrived at a compromise agreement that the value of the said improvement/building is P400,000.00 The Court notes that the plaintiff has already receivedP300,000.00. However, when

defendant was ready to pay the balance of P100,000.00, the plaintiff now insists that there should be a rental to be paid by

defendants. Whether or not this should be paid by defendants, incident is hereby scheduled for hearing on November 12,

1997 at 8:30 a.m.

Meantime, defendants are directed to pay plaintiff the balance of P100,000.00.

SO ORDERED.[15]

On December 1997, after paying the said P100,000 balance to Pedro Pecson the spousesNuguid prayed for the closure and

termination of the case, as well as the cancellation of the notice of lis pendens on the title of the property on the ground that Pedro

Pecson’s claim for rentals was devoid of factual and legal bases.[16]

After conducting a hearing, the lower court issued an Order dated July 31, 1998, directing the spouses to pay the sum

of P1,344,000 as reimbursement of the unrealized income of Pecson for the period beginning November 22, 1993 up to December

1997. The sum was based on the computation of P28,000/month rentals of the four-door apartment, thus:

The Court finds plaintiff’s motion valid and meritorious. The decision of the Supreme Court in the aforesaid case [Pecson vs. Court of Appeals, 244 SCRA 407] which set aside the Order of this Court of November 15, 1993 has in effect upheld plaintiff’s right of

possession of the building for as long as he is not fully paid the value thereof. It follows, as declared by the Supreme Court in said

decision that the plaintiff is entitled to the income derived therefrom, thus –

. . .

Records show that the plaintiff was dispossessed of the premises on November 22, 1993 and that he was fully paid the value of his

building in December 1997. Therefore, he is entitled to the income thereof beginning on November 22, 1993, the time he was

dispossessed, up to the time of said full payment, in December 1997, or a total of 48 months.

The only question left is the determination of income of the four units of apartments per month. But as correctly pointed out by

plaintiff, the defendants have themselves submitted their affidavits attesting that the income derived from three of the four units of the apartment building is P21,000.00 or P7,000.00 each per month, or P28,000.00 per month for the whole four units. Hence, at

P28,000.00 per month, multiplied by 48 months, plaintiff is entitled to be paid by defendants the amount of P1,344,000.00.[17]

The Nuguid spouses filed a motion for reconsideration but this was denied for lack of merit.[18]

The Nuguid couple then appealed the trial court’s ruling to the Court of Appeals, their action docketed as CA-G.R. CV No.

64295.

32

In the Court of Appeals, the order appealed from in CA-G.R. CV No. 64295, was modified. The CA reduced the rentals

from P1,344,000 to P280,000 in favor of the appellee.[19] The said amount represents accrued rentals from the determination of the

current market value on January 31, 1997[20] until its full payment on December 12, 1997.

Hence, petitioners state the sole assignment of error now before us as follows:

THE COURT OF APPEALS ERRED IN HOLDING PETITIONERS LIABLE TO PAY RENT OVER AND ABOVE THE CURRENT MARKET VALUE OF THE IMPROVEMENT WHEN SUCH WAS NOT PROVIDED FOR IN THE DISPOSITIVE PORTION OF THE SUPREME COURT’S RULING

IN G.R. No. 115814.

Petitioners call our attention to the fact that after reaching an agreed price of P400,000 for the improvements, they only made a partial payment of P300,000. Thus, they contend that their failure to pay the full price for the improvements will, at most,

entitle respondent to be restored to possession, but not to collect any rentals. Petitioners insist that this is the proper

interpretation of the dispositive portion of the decision in G.R. No. 115814, which states in part that “[t]he value so determined

shall be forthwith paid by the private respondents [Spouses Juan and Erlinda Nuguid] to the petitioner [Pedro

Pecson] otherwise the petitioner shall be restored to the possession of the apartment building until payment of the required

indemnity.”[21]

Now herein respondent, Pecson, disagrees with herein petitioners’ contention. He argues that petitioners are wrong in

claiming that inasmuch as his claim for rentals was not determined in the dispositive portion of the decision in G.R. No. 115814, it

could not be the subject of execution. He points out that in moving for an accounting, all he asked was that the value of the fruits

of the property during the period he was dispossessed be accounted for, since this Court explicitly recognized in G.R. No. 115814, he was entitled to the property. He points out that this Court ruled that “[t]he petitioner [Pecson] not having been so paid, he was

entitled to retain ownership of the building and, necessarily, the income therefrom.”[22] In other words, says respondent, accounting

was necessary. For accordingly, he was entitled to rental income from the property. This should be given effect. The Court could

have very well specifically included rent (as fruit or income of the property), but could not have done so at the time the Court

pronounced judgment because its value had yet to be determined, according to him. Additionally, he faults the appellate court for

modifying the order of the RTC, thus defeating his right as a builder in good faith entitled to rental from the period of his

dispossession to full payment of the price of his improvements, which spans from November 22, 1993 to December 1997, or a

period of more than four years.

It is not disputed that the construction of the four-door two-storey apartment, subject of this dispute, was undertaken at the time when Pecson was still the owner of the lot. When the Nuguids became the uncontested owner of the lot on June 23, 1993, by

virtue of entry of judgment of the Court’s decision, dated May 25, 1993, in G.R. No. 105360, the apartment building was already in

existence and occupied by tenants. In its decision dated May 26, 1995 in G.R. No. 115814, the Court declared the rights and

obligations of the litigants in accordance with Articles 448 and 546 of the Civil Code. These provisions of the Code are directly

applicable to the instant case.

Under Article 448, the landowner is given the option, either to appropriate the improvement as his own upon payment of the

proper amount of indemnity or to sell the land to the possessor in good faith. Relatedly, Article 546 provides that a builder in good

faith is entitled to full reimbursement for all the necessary and useful expenses incurred; it also gives him right of retention until full

reimbursement is made.

While the law aims to concentrate in one person the ownership of the land and the improvements thereon in view of the impracticability of creating a state of forced co-ownership,[23]it guards against unjust enrichment insofar as the good-faith builder’s

improvements are concerned. The right of retention is considered as one of the measures devised by the law for the protection of

builders in good faith. Its object is to guarantee full and prompt reimbursement as it permits the actual possessor to remain in

possession while he has not been reimbursed (by the person who defeated him in the case for possession of the property) for those

necessary expenses and useful improvements made by him on the thing possessed.[24] Accordingly, a builder in good faith cannot

be compelled to pay rentals during the period of retention[25] nor be disturbed in his possession by ordering him to vacate. In

addition, as in this case, the owner of the land is prohibited from offsetting or compensating the necessary and useful expenses

with the fruits received by the builder-possessor in good faith. Otherwise, the security provided by law would be impaired. This is

so because the right to the expenses and the right to the fruits both pertain to the possessor, making compensation juridically impossible; and one cannot be used to reduce the other.[26]

As we earlier held, since petitioners opted to appropriate the improvement for themselves as early as June 1993, when they

applied for a writ of execution despite knowledge that the auction sale did not include the apartment building, they could not benefit

from the lot’s improvement, until they reimbursed the improver in full, based on the current market value of the property.

Despite the Court’s recognition of Pecson’s right of ownership over the apartment building, the petitioners still insisted on

dispossessing Pecson by filing for a Writ of Possession to cover both the lot and the building. Clearly, this resulted in a violation of

respondent’s right of retention. Worse, petitioners took advantage of the situation to benefit from the highly valued, income-

yielding, four-unit apartment building by collecting rentals thereon, before they paid for the cost of the apartment building. It was

only four years later that they finally paid its full value to the respondent.

Petitioners’ interpretation of our holding in G.R. No. 115814 has neither factual nor legal basis. The decision of May 26, 1995, should be construed in connection with the legal principles which form the basis of the decision, guided by the precept that

judgments are to have a reasonable intendment to do justice and avoid wrong.[27]

The text of the decision in G.R. No. 115814 expressly exempted Pecson from liability to pay rentals, for we found that the

Court of Appeals erred not only in upholding the trial court’s determination of the indemnity, but also in ordering him to account for

the rentals of the apartment building from June 23, 1993 to September 23, 1993, the period from entry of judgment

untilPecson’s dispossession. As pointed out by Pecson, the dispositive portion of our decision in G.R. No. 115814 need not

specifically include the income derived from the improvement in order to entitle him, as a builder in good faith, to such

income. The right of retention, which entitles the builder in good faith to the possession as well as the income derived therefrom, is

already provided for under Article 546 of the Civil Code.

33

Given the circumstances of the instant case where the builder in good faith has been clearly denied his right of retention for

almost half a decade, we find that the increased award of rentals by the RTC was reasonable and equitable. The petitioners had

reaped all the benefits from the improvement introduced by the respondent during said period, without paying any amount to the

latter as reimbursement for his construction costs and expenses. They should account and pay for such benefits.

We need not belabor now the appellate court’s recognition of herein respondent’s entitlement to rentals from the date of the determination of the current market value until its full payment. Respondent is clearly entitled to payment by virtue of his right of

retention over the said improvement.

WHEREFORE, the instant petition is DENIED for lack of merit. The Decision dated May 21, 2001 of the Court of Appeals in

CA-G.R. CV No. 64295 is SET ASIDE and the Order dated July 31, 1998, of the Regional Trial Court, Branch 101, Quezon City, in

Civil Case No. Q-41470 ordering the herein petitioners, Spouses Juan and Erlinda Nuguid, to account for the rental income of the

four-door two-storey apartment building from November 1993 until December 1997, in the amount of P1,344,000, computed on

the basis of Twenty-eight Thousand (P28,000.00) pesos monthly, for a period of 48 months, is hereby REINSTATED. Until fully

paid, said amount of rentals should bear the legal rate of interest set at six percent (6%) per annum computed from the date of

RTC judgment. If any portion thereof shall thereafter remain unpaid, despite notice of finality of this Court’s judgment, said

remaining unpaid amount shall bear the rate of interest set at twelve percent (12%) per annum computed from the date of said notice. Costs against petitioners.

SO ORDERED.

IGNACIO GRANDE, ET AL., petitioners, vs.

HON. COURT OF APPEALS, DOMINGO CALALUNG, and ESTEBAN CALALUNG, respondents.

Bartolome Guirao and Antonio M. Orara for petitioners.

Gonzales and Fernandez for respondents.

BARRERA, J.:

This is an appeal taken by petitioners Ignacio, Eulogia, Alfonso, Eulalia, and Sofia Grande, from the decision of the Court of Appeals

(CA-G.R. No. 25169-R) reversing that of the Court of First Instance of Isabela (Civil Case No. 1171), and dismissing petitioners'

action against respondents Domingo and Esteban Calalung, to quiet title to and recover possession of a parcel of land allegedly

occupied by the latter without petitioners' consent.

The facts of the case, which are undisputed, briefly are: Petitioners are the owners of a parcel of land, with an area of 3.5032

hectares, located at barrio Ragan, municipality of Magsaysay (formerly Tumauini), province of Isabela, by inheritance from their

deceased mother Patricia Angui (who inherited it from her parents Isidro Angui and Ana Lopez, in whose name said land appears

registered, as shown by Original Certificate of Title No. 2982, issued on June 9, 1934). Said property is identified as Lot No. 1, Plan

PSU-83342. When it was surveyed for purposes of registration sometime in 1930, its northeastern boundary was the Cagayan River

(the same boundary stated in the title). Since then, and for many years thereafter, a gradual accretion on the northeastern side took place, by action of the current of the Cagayan River, so much so, that by 1958, the bank thereof had receded to a distance of

about 105 meters from its original site, and an alluvial deposit of 19,964 square meters (1.9964 hectares), more or less, had been

added to the registered area (Exh. C-1).

On January 25, 1958, petitioners instituted the present action in the Court of First Instance of Isabela against respondents, to quiet

title to said portion (19,964 square meters) formed by accretion, alleging in their complaint (docketed as Civil Case No. 1171) that

they and their predecessors-in-interest, were formerly in peaceful and continuous possession thereof, until September, 1948, when respondents entered upon the land under claim of ownership. Petitioners also asked for damages corresponding to the value of the

fruits of the land as well as attorney's fees and costs. In their answer (dated February 18, 1958), respondents claim ownership in

themselves, asserting that they have been in continuous, open, and undisturbed possession of said portion, since prior to the year

1933 to the present.

After trial, the Court of First Instance of Isabela, on May 4, 1959, rendered a decision adjudging the ownership of the portion in

question to petitioners, and ordering respondents to vacate the premises and deliver possession thereof to petitioners, and to pay to the latter P250.00 as damages and costs. Said decision, in part, reads:

It is admitted by the parties that the land involved in this action was formed by the gradual deposit of alluvium brought

about by the action of the Cagayan River, a navigable river. We are inclined to believe that the accretion was formed on

the northeastern side of the land covered by Original Certificate of Title No. 2982 after the survey of the registered land

in 1931, because the surveyors found out that the northeastern boundary of the land surveyed by them was the Cagayan

River, and not the land in question. Which is indicative of the fact that the accretion has not yet started or begun in 1931.

And, as declared by Pedro Laman, defendant witness and the boundary owner on the northwest of the registered land of the plaintiffs, the accretion was a little more than one hectare, including the stony portion, in 1940 or 1941. Therefore,

the declarations of the defendant Domingo Calalung and his witness, Vicente C. Bacani, to the effect that the land in

question was formed by accretion since 1933 do not only contradict the testimony of defendants' witness Pedro Laman,

but could not overthrow the incontestable fact that the accretion with an area of 4 hectare more or less, was formed in

1948, reason for which, it was only declared in that same year for taxation purposes by the defendants under Tax Dec.

No. 257 (Exh. "2") when they entered upon the land. We could not give credence to defendants' assertion that Tax Dec.

No. 257 (Exh. "2") cancelled Tax Dee. No. 28226 (Exh. "1"), because Exh. "2" says that "tax under this declaration

begins with the year 1948. But, the fact that defendants declared the land for taxation purposes since 1948, does not

34

mean that they become the owner of the land by mere occupancy, for it is a new provision of the New Civil Code that

ownership of a piece of land cannot be acquired by occupation (Art. 714, New Civil Code). The land in question being an

accretion to the mother or registered land of the plaintiffs, the accretion belongs to the plaintiffs (Art. 457, New Civil

Code; Art. 366, Old Civil Code). Assuming arguendo, that the accretion has been occupied by the defendants since 1948,

or earlier, is of no moment, because the law does not require any act of possession on the part of the owner of the riparian owner, from the moment the deposit becomes manifest (Roxas v. Tuason, 9 Phil. 408; Cortez v. City of Manila,

10 Phil. 567). Further, no act of appropriation on the part of the reparian owner is necessary, in order to acquire

ownership of the alluvial formation, as the law does not require the same (3 Manresa, C.C., pp. 321-326).

This brings us now to the determination of whether the defendants, granting that they have been in possession of the

alluvium since 1948, could have acquired the property by prescription. Assuming that they occupied the land in

September, 1948, but considering that the action was commenced on January 25, 1958, they have not been in

possession of the land for ten (10) years; hence, they could not have acquired the land by ordinary prescription (Arts. 1134 and 1138, New Civil Code). Moreover, as the alluvium is, by law, part and parcel of the registered property, the

same may be considered as registered property, within the meaning of Section 46 of Act No. 496: and, therefore, it could

not be acquired by prescription or adverse possession by another person.

Unsatisfied, respondents appealed to the Court of Appeals, which rendered, on September 14, 1960, the decision adverted to at the

beginning of this opinion, partly stating:

That the area in controversy has been formed through a gradual process of alluvium, which started in the early thirties, is

a fact conclusively established by the evidence for both parties. By law, therefore, unless some superior title has

supervened, it should properly belong to the riparian owners, specifically in accordance with the rule of natural accession

in Article 366 of the old Civil Code (now Article 457), which provides that "to the owner of lands adjoining the banks of

rivers, belongs the accretion which they gradually receive from the effects of the current of the waters." The defendants,

however, contend that they have acquired ownership through prescription. This contention poses the real issue in this

case. The Courta quo, has resolved it in favor of the plaintiffs, on two grounds: First, since by accession, the land in question pertains to the original estate, and since in this instance the original estate is registered, the accretion,

consequently, falls within the purview of Section 46 of Act No. 496, which states that "no title to registered land in

derogation to that of the registered owner shall be acquired by prescription or adverse possession"; and, second, the

adverse possession of the defendant began only in the month of September, 1948, or less than the 10-year period

required for prescription before the present action was instituted.

As a legal proposition, the first ground relied upon by the trial court, is not quite correct. An accretion to registered land, while declared by specific provision of the Civil Code to belong to the owner of the land as a natural accession thereof,

does not ipso jure become entitled to the protection of the rule of imprescriptibility of title established by the Land

Registration Act. Such protection does not extend beyond the area given and described in the certificate. To hold

otherwise, would be productive of confusion. It would virtually deprive the title, and the technical description of the land

given therein, of their character of conclusiveness as to the identity and area of the land that is registered. Just as the

Supreme Court, albeit in a negative manner, has stated that registration does not protect the riparian owner against the

erosion of the area of his land through gradual changes in the course of the adjoining stream (Payatas Estate

Development Co. v. Tuason, 53 Phil. 55), so registration does not entitle him to all the rights conferred by Land

Registration Act, in so far as the area added by accretion is concerned. What rights he has, are declared not by said Act,

but by the provisions of the Civil Code on accession: and these provisions do not preclude acquisition of the addition area by another person through prescription. This Court has held as much in the case of Galindez, et al. v. Baguisa, et al., CA-

G.R. No. 19249-R, July 17, 1959.

We now proposed to review the second ground relied upon by the trial court, regarding the length of time that the

defendants have been in possession. Domingo Calalung testified that he occupied the land in question for the first time in

1934, not in 1948 as claimed by the plaintiffs. The area under occupancy gradually increased as the years went by. In

1946, he declared the land for purposes of taxation (Exhibit 1). This tax declaration was superseded in 1948 by another (Exhibit 2), after the name of the municipality wherein it is located was changed from Tumauini to Magsaysay. Calalung's

testimony is corroborated by two witnesses, both owners of properties nearby. Pedro Laman, 72 years of age, who was

Municipal president of Tumauini for three terms, said that the land in question adjoins his own on the south, and that

since 1940 or 1951, he has always known it to be in the peaceful possession of the defendants. Vicente C. Bacani

testified to the same effect, although, he said that the defendants' possession started sometime in 1933 or 1934. The

area thereof, he said, was then less than one hectare.

We find the testimony of the said witnesses entitled to much greater weight and credence than that of the plaintiff Pedro Grande and his lone witness, Laureana Rodriguez. The first stated that the defendants occupied the land in question only

in 1948; that he called the latter's attention to the fact that the land was his, but the defendants, in turn, claimed that

they were the owners, that the plaintiffs did not file an action until 1958, because it was only then that they were able to

obtain the certificate of title from the surveyor, Domingo Parlan; and that they never declared the land in question for

taxation purposes or paid the taxes thereon. Pedro Grande admitted that the defendants had the said land surveyed in

April, 1958, and that he tried to stop it, not because he claimed the accretion for himself and his co-plaintiffs, but

because the survey included a portion of the property covered by their title. This last fact is conceded by the defendants

who, accordingly, relinquished their possession to the part thus included, containing an area of some 458 square

meters.1äwphï1.ñët

The oral evidence for the defendants concerning the period of their possession — from 1933 to 1958 — is not only

preponderant in itself, but is, moreover, supported by the fact that it is they and not the plaintiffs who declared the

35

disputed property for taxation, and by the additional circumstance that if the plaintiff had really been in prior possession

and were deprived thereof in 1948, they would have immediately taken steps to recover the same. The excuse they gave

for not doing so, namely, that they did not receive their copy of the certificate of title to their property until 1958 for lack

of funds to pay the fees of the surveyor Domingo Parlan, is too flimsy to merit any serious consideration. The payment of

the surveyor's fees had nothing to do with their right to obtain a copy of the certificate. Besides, it was not necessary for them to have it in their hands, in order to file an action to recover the land which was legally theirs by accession and of

which, as they allege, they had been illegally deprived by the defendants. We are convinced, upon consideration of the

evidence, that the latter, were really in possession since 1934, immediately after the process of alluvion started, and that

the plaintiffs woke up to their rights only when they received their copy of the title in 1958. By then, however,

prescription had already supervened in favor of the defendants.

It is this decision of the Court of Appeals which petitioners seek to be reviewed by us.

The sole issue for resolution in this case is whether respondents have acquired the alluvial property in question through

prescription.

There can be no dispute that both under Article 457 of the New Civil Code and Article 366 of the old, petitioners are the lawful

owners of said alluvial property, as they are the registered owners of the land which it adjoins. The question is whether the accretion becomes automatically registered land just because the lot which receives it is covered by a Torrens title thereby making

the alluvial property imprescriptible. We agree with the Court of Appeals that it does not, just as an unregistered land purchased by

the registered owner of the adjoining land does not, by extension, become ipso facto registered land. Ownership of a piece of land

is one thing, and registration under the Torrens system of that ownership is quite another. Ownership over the accretion received

by the land adjoining a river is governed by the Civil Code. Imprescriptibility of registered land is provided in the registration law.

Registration under the Land Registration and Cadastral Acts does not vest or give title to the land, but merely confirms and

thereafter protects the title already possessed by the owner, making it imprescriptible by occupation of third parties. But to obtain

this protection, the land must be placed under the operation of the registration laws wherein certain judicial procedures have been

provided. The fact remain, however, that petitioners never sought registration of said alluvial property (which was formed sometime after petitioners' property covered by Original Certificate of Title No. 2982 was registered on June 9, 1934) up to the time

they instituted the present action in the Court of First Instance of Isabela in 1958. The increment, therefore, never became

registered property, and hence is not entitled or subject to the protection of imprescriptibility enjoyed by registered property under

the Torrens system. Consequently, it was subject to acquisition through prescription by third persons.

The next issue is, did respondents acquire said alluvial property through acquisitive prescription? This is a question which requires

determination of facts: physical possession and dates or duration of such possession. The Court of Appeals, after analyzing the evidence, found that respondents-appellees were in possession of the alluvial lot since 1933 or 1934, openly, continuously and

adversely, under a claim of ownership up to the filing of the action in 1958. This finding of the existence of these facts, arrived at

by the Court of Appeals after an examination of the evidence presented by the parties, is conclusive as to them and can not be

reviewed by us.

The law on prescription applicable to the case is that provided in Act 190 and not the provisions of the Civil Code, since the

possession started in 1933 or 1934 when the pertinent articles of the old Civil Code were not in force and before the effectivity of

the new Civil Code in 1950. Hence, the conclusion of the Court of Appeals that the respondents acquired alluvial lot in question by acquisitive prescription is in accordance with law.

The decision of the Court of Appeals under review is hereby affirmed, with costs against the petitioners. So ordered.

Bengzon, C.J., Padilla, Bautista Angelo, Labrador, Concepcion, Paredes and Dizon, JJ., concur.

Reyes, J.B.L., Regala and Makalintal, JJ., took no part.

SIMPLICIO BINALAY, PONCIANO GANNABAN, NICANOR MACUTAY, DOMINGO ROSALES, GREGORIO ARGONZA, EUSTAQUIO BAUA, FLORENTINO ROSALES, TEODORO MABBORANG, PATRICIO MABBORANG and FULGENCIO

MORA, petitioners

vs.

GUILLERMO MANALO and COURT OF APPEALS, respondents.

Josefin De Alban Law Office for Petitioners.

FELICIANO, J.:p

The late Judge Taccad originally owned a parcel of land situated in Tumauini, Isabela having an estimated area of twenty (20)

hectares. The western portion of this land bordering on the Cagayan River has an elevation lower than that of the eastern portion

which borders on the national road. Through the years, the western portion would periodically go under the waters of the Cagayan

River as those waters swelled with the coming of the rains. The submerged portion, however, would re-appear during the dry

season from January to August. It would remain under water for the rest of the year, that is, from September to December during

the rainy season.

36

The ownership of the landholding eventually moved from one person to another. On 9 May 1959, respondent Guillermo Manalo

acquired 8.65 hectares thereof from Faustina Taccad, daughter of Judge Juan Taccad. The land sold was described in the Deed of

Absolute Sale 1 as follows:

. . . a parcel of agricultural land in Balug, Tumauini, Isabela, containing an area of 8.6500 hectares, more or

less; bounded on the North by Francisco Forto on the East by National Road; on South by Julian Tumolva and

on the West by Cagayan River; declared for taxation under Tax Declaration No. 12681 in the name of Faustina

Taccad, and assessed at P 750.00. . . .

Later in 1964, respondent Manalo purchased another 1.80 hectares from Gregorio Taguba who had earlier acquired the same from Judge Juan Taccad. The second purchase brought the total acquisition of respondent Manalo to 10.45 hectares. The second piece of

property was more particularly described as follows:

. . . a piece of agricultural land consisting of tobacco land, and containing an area of 18,000 square meters,

more or less, bounded on the North by Balug Creek; on the South, by Faustina Taccad (now Guillermo R.

Manalo); on the East, by a Provincial Road; and on the West, by Cagayan River assessed at P 440.00, as tax

Declaration No. 3152. . . . 2

During the cadastral survey conducted at Balug, Tumauini, Isabela on 21 October 1969, the two (2) parcels of land belonging to

respondent Manalo were surveyed and consolidated into one lot, designated as Lot No. 307, Pls-964. Lot 307 which contains 4.6489

hectares includes: (a) the whole of the 1.80 hectares acquired from Gregorio Taguba; and (b) 2.8489 hectares out of the 8.65

hectares purchased from Faustina Taccad. As the survey was conducted on a rainy month, a portion of the land bought from

Faustina Taccad then under water was left unsurveyed and was not included in Lot 307.

The Sketch Plan 3 submitted during the trial of this case and which was identified by respondent Manalo shows that the Cagayan

River running from south to north, forks at a certain point to form two (2) branches—the western and the eastern branches—and

then unites at the other end, further north, to form a narrow strip of land. The eastern branch of the river cuts through the land of

respondent Manalo and is inundated with water only during the rainy season. The bed of the eastern branch is the submerged or

the unsurveyed portion of the land belonging to respondent Manalo. For about eight (8) months of the year when the level of water

at the point where the Cagayan River forks is at its ordinary depth, river water does not flow into the eastern branch. While this

condition persists, the eastern bed is dry and is susceptible to cultivation.

Considering that water flowed through the eastern branch of the Cagayan River when the cadastral survey was conducted, the

elongated strip of land formed by the western and the eastern branches of the Cagayan River looked very much like an island. This

strip of land was surveyed on 12 December 1969. 4 It was found to have a total area of 22.7209 hectares and was designated as

Lot 821 and Lot 822. The area of Lot 822 is 10.8122 hectares while Lot 821 has an area of 11.9087 hectares. Lot 821 is located

directly opposite Lot 307 and is separated from the latter only by the eastern branch of the Cagayan River during the rainy season

and, during the dry season, by the exposed, dry river bed, being a portion of the land bought from Faustina Taccad. Respondent

Manalo claims that Lot 821 also belongs to him by way of accretion to the submerged portion of the property to which it is adjacent.

Petitioners who are in possession of Lot 821, upon the other hand, insist that they own Lot 821. They occupy the outer edges of Lot

821 along the river banks, i.e., the fertile portions on which they plant tobacco and other agricultural products. They also cultivate

the western strip of the unsurveyed portion during summer. 5 This situation compelled respondent Manalo to file a case for forcible

entry against petitioners on 20 May 1969. The case was dismissed by the Municipal Court of Tumauini, Isabela for failure of both

parties to appear. On 15 December 1972, respondent Manalo again filed a case for forcible entry against petitioners. The latter case

was similarly dismissed for lack of jurisdiction by the Municipal Court of Tumauini, Isabela.

On 24 July 1974, respondent Manalo filed a complaints 6 before the then Court of First Instance of Isabela, Branch 3 for quieting of

title, possession and damages against petitioners. He alleged ownership of the two (2) parcels of land he bought separately from

Faustina Taccad and Gregorio Taguba for which reason he prayed that judgment be entered ordering petitioners to vacate the

western strip of the unsurveyed portion. Respondent Manalo likewise prayed that judgment be entered declaring him as owner of

Lot 821 on which he had laid his claim during the survey.

Petitioners filed their answer denying the material allegations of the complaint. The case was then set for trial for failure of the

parties to reach an amicable agreement or to enter into a stipulation of facts. 7 On 10 November 1982, the trial court rendered a

decision with the following dispositive portion:

WHEREFORE, in the light of the foregoing premises, the Court renders judgment against the defendants and in

favor of the plaintiff and orders:

1. That plaintiff, Guillermo Manalo, is declared the lawful owner of the land in question, Lot No. 821, Pls-964 of

Tumauini Cadastre, and which is more particularly described in paragraph 2-b of the Complaint;

2. That the defendants are hereby ordered to vacate the premises of the land in question, Lot No. 821, Pls-964

of Tumauini Cadastre, and which is more particularly described in paragraph 2-b of the Complaint;

37

3. That the defendants are being restrained from entering the premises of the land in question, Lot No. 821,

Pls-964 of Tumauini Cadastre, and which is more particularly described in paragraph 2-b of the Complaint; and

4. That there is no pronouncement as to attorney's fees and costs.

SO ORDERED. 8

Petitioners appealed to the Court of Appeals which, however, affirmed the decision of the trial court. They filed a motion for reconsideration, without success.

While petitioners insist that Lot 821 is part of an island surrounded by the two (2) branches of the Cagayan River, the Court of

Appeals found otherwise. The Court of Appeals concurred with the finding of the trial court that Lot 821 cannot be considered

separate and distinct from Lot 307 since the eastern branch of the Cagayan River substantially dries up for the most part of the

year such that when this happens, Lot 821 becomes physically (i.e., by land) connected with the dried up bed owned by respondent

Manalo. Both courts below in effect rejected the assertion of petitioners that the depression on the earth's surface which separates

Lot 307 and Lot 821 is, during part of the year, the bed of the eastern branch of the Cagayan River.

It is a familiar rule that the findings of facts of the trial court are entitled to great respect, and that they carry even more weight

when affirmed by the Court of Appeals. 9 This is in recognition of the peculiar advantage on the part of the trial court of being able

to observe first-hand the deportment of the witnesses while testifying. Jurisprudence is likewise settled that the Court of Appeals is

the final arbiter of questions of fact. 10 But whether a conclusion drawn from such findings of facts is correct, is a question of law

cognizable by this Court. 11

In the instant case, the conclusion reached by both courts below apparently collides with their findings that periodically at the onset

of and during the rainy season, river water flows through the eastern bed of the Cagayan River. The trial court held:

The Court believes that the land in controversy is of the nature and character of alluvion (Accretion), for it

appears that during the dry season, the body of water separating the same land in controversy (Lot No. 821, Pls-964) and the two (2) parcels of land which the plaintiff purchased from Gregorio Taguba and Justina Taccad

Cayaba becomes a marshy land and is only six (6) inches deep and twelve (12) meters in width at its widest in

the northern tip (Exhs. "W", "W-l", "W-2", "W-3" and "W-4"), It has been held by our Supreme Court that "the

owner of the riparian land which receives the gradual deposits of alluvion, does not have to make an express

act of possession. The law does not require it, and the deposit created by the current of the water becomes

manifest" (Roxas vs. Tuazon, 6 Phil. 408). 12

The Court of Appeals adhered substantially to the conclusion reached by the trial court, thus:

As found by the trial court, the disputed property is not an island in the strict sense of the word since the

eastern portion of the said property claimed by appellants to be part of the Cagayan River dries up during

summer. Admittedly, it is the action of the heavy rains which comes during rainy season especially from

September to November which increases the water level of the Cagayan river. As the river becomes swollen

due to heavy rains, the lower portion of the said strip of land located at its southernmost point would be

inundated with water. This is where the water of the Cagayan river gains its entry. Consequently, if the water level is high the whole strip of land would be under water.

In Government of the Philippine Islands vs. Colegio de San Jose, it was held that —

According to the foregoing definition of the words "ordinary" and "extra-ordinary," the highest depth of the waters of Laguna de Bay during the dry season is the ordinary one,

and the highest depth they attain during the extra-ordinary one (sic); inasmuch as the

former is the one which is regular, common, natural, which occurs always or most of the

time during the year, while the latter is uncommon, transcends the general rule, order and

measure, and goes beyond that which is the ordinary depth. If according to the definition

given by Article 74 of the Law of Waters quoted above, the natural bed or basin of the

lakes is the ground covered by their waters when at their highest ordinary depth, the

natural bed or basin of Laguna de Bay is the ground covered by its waters when at their

highest depth during the dry season, that is up to the northeastern boundary of the two parcels of land in question.

We find the foregoing ruling to be analogous to the case at bar. The highest ordinary level of the waters of the

Cagayan River is that attained during the dry season which is confined only on the west side of Lot [821] and

Lot [822]. This is the natural Cagayan river itself. The small residual of water between Lot [821] and 307 is

part of the small stream already in existence when the whole of the late Judge Juan Taccad's property was still

susceptible to cultivation and uneroded. 13

The Court is unable to agree with the Court of Appeals that Government of the Philippine Islands vs. Colegio de San Jose 14 is

applicable to the present case. That case involved Laguna de Bay; since Laguna de Bay is a lake, the Court applied the legal

38

provisions governing the ownership and use of lakes and their beds and shores, in order to determine the character and ownership

of the disputed property. Specifically, the Court applied the definition of the natural bed or basin of lakes found in Article 74 of the

Law of Waters of 3 August 1866. Upon the other hand, what is involved in the instant case is the eastern bed of the Cagayan River.

We believe and so hold that Article 70 of the Law of Waters of 3 August 1866 is the law applicable to the case at bar:

Art. 70. The natural bed or channel of a creek or river is the ground covered by its waters during the highest

floods. (Emphasis supplied)

We note that Article 70 defines the natural bed or channel of a creek or river as the ground covered by its waters during

the highest floods. The highest floods in the eastern branch of the Cagayan River occur with the annual coming of the

rains as the river waters in their onward course cover the entire depressed portion. Though the eastern bed substantially

dries up for the most part of the year (i.e., from January to August), we cannot ignore the periodical swelling of the

waters ( i.e., from September to December) causing the eastern bed to be covered with flowing river waters.

The conclusion of this Court that the depressed portion is a river bed rests upon evidence of record. Firstly, respondent Manalo admitted in open court that the entire area he bought from Gregorio Taguba was included in Lot 307. 15 If the 1.80 hectares

purchased from Gregorio Taguba was included in Lot 307, then the Cagayan River referred to as the western boundary in the Deed

of Sale transferring the land from Gregorio Taguba to respondent Manalo as well as the Deed of Sale signed by Faustina Taccad,

must refer to the dried up bed (during the dry months) or the eastern branch of the river (during the rainy months). In the Sketch

Plan attached to the records of the case, Lot 307 is separated from the western branch of the Cagayan River by a large tract of land

which includes not only Lot 821 but also what this Court characterizes as the eastern branch of the Cagayan River.

Secondly, the pictures identified by respondent Manalo during his direct examination depict the depressed portion as a river bed.

The pictures, marked as Exhibits "W" to "W-4", were taken in July 1973 or at a time when the eastern bed becomes visible. 16 Thus,

Exhibit "W-2" which according to respondent Manalo was taken facing the east and Exhibit "W-3" which was taken facing the west

both show that the visible, dried up portion has a markedly lower elevation than Lot 307 and Lot 821. It has dike-like slopes on

both sides connecting it to Lot 307 and Lot 821 that are vertical upward and very prominent. This topographic feature is compatible

with the fact that a huge volume of water passes through the eastern bed regularly during the rainy season. In addition, petitioner

Ponciano Gannaban testified that one had to go down what he called a "cliff" from the surveyed portion of the land of respondent

Manalo to the depressed portion. The cliff, as related by petitioner Gannaban, has a height of eight (8) meters. 17

The records do not show when the Cagayan River began to carve its eastern channel on the surface of the earth. However, Exhibit

"E" 18 for the prosecution which was the Declaration of Real Property standing in the name of Faustina Taccad indicates that the

eastern bed already existed even before the sale to respondent Manalo. The words "old bed" enclosed in parentheses—perhaps

written to make legitimate the claim of private ownership over the submerged portion—is an implied admission of the existence of

the river bed. In the Declaration of Real Property made by respondent Manalo, the depressed portion assumed the name Rio Muerte

de Cagayan. Indeed, the steep dike-like slopes on either side of the eastern bed could have been formed only after a prolonged

period of time.

Now, then, pursuant to Article 420 of the Civil Code, respondent Manalo did not acquire private ownership of the bed of the eastern

branch of the river even if it was included in the deeds of absolute sale executed by Gregorio Taguba and Faustina Taccad in his

favor. These vendors could not have validly sold land that constituted property of public dominion. Article 420 of the Civil Code

states:

The following things are property of public dominion:

(1) Those intended for public use, such as roads, canals, rivers, torrents, ports and bridges constructed by the

State, banks, shores, roadsteads, and others of similar character;

(2) Those which belong to the State, without being for public use, and are intended for some public service or

for the development of the national wealth. (Emphasis supplied)

Although Article 420 speaks only of rivers and banks, "rivers" is a composite term which includes: (1) the running waters, (2) the

bed, and (3) the banks. 19 Manresa, in commenting upon Article 339 of the Spanish Civil Code of 1889 from which Article 420 of the

Philippine Civil Code was taken, stressed the public ownership of river beds:

La naturaleza especial de los rios, en punto a su disfrute general, hace que sea necesario considerar en su

relacion de dominio algo mas que sus aguas corrientes. En efecto en todo rio es preciso distinguir 1. esta agua

corriente; 2. el alveo o cauce, y 3. las riberas. Ahora bien: son estas dos ultimas cosas siempre de dominio

publico, como las aguas?

Realmente no puede imaginarse un rio sin alveo y sin ribera; de suerte que al decir el Codigo civil que los rios

son de dominio publico, parece que debe ir implicito el dominio publico de aquellos tres elementos que integran

el rio. Por otra parte, en cuanto a los alveos o cauces tenemos la declaracion del art. 407, num 1, donde

dice: son de dominion publico . . . los rios y sus cauces naturales; declaracion que concuerda con lo que

dispone el art. 34 de la ley de [Aguas], segun el cual, son de dominion publico: 1. los alveos o cauces de los

39

arroyos que no se hallen comprendidos en el art. 33, y 2. los alveos o cauces naturales de los rios en la

extension que cubran sus aguas en las mayores crecidas ordinarias. 20 (Emphasis supplied)

The claim of ownership of respondent Manalo over the submerged portion is bereft of basis even if it were alleged and proved that

the Cagayan River first began to encroach on his property after the purchase from Gregorio Taguba and Faustina Taccad. Article

462 of the Civil Code would then apply divesting, by operation of law, respondent Manalo of private ownership over the new river

bed. The intrusion of the eastern branch of the Cagayan River into his landholding obviously prejudiced respondent Manalo but this

is a common occurrence since estates bordering on rivers are exposed to floods and other evils produced by the destructive force of

the waters. That loss is compensated by, inter alia, the right of accretion acknowledged by Article 457 of the Civil Code. 21 It so

happened that instead of increasing the size of Lot 307, the eastern branch of the Cagayan River had carved a channel on it.

We turn next to the issue of accretion. After examining the records of the case, the Court considers that there was no evidence to

prove that Lot 821 is an increment to Lot 307 and the bed of the eastern branch of the river. Accretion as a mode of acquiring

property under Article 457 of the Civil Code requires the concurrence of three (3) requisites: (a) that the deposition of soil or

sediment be gradual and imperceptible; (b) that it be the result of the action of the waters of the river (or sea); and (c) that the

land where accretion takes place is adjacent to the banks of rivers (or the sea coast). 22 The Court notes that the parcels of land

bought by respondent Manalo border on the eastern branch of the Cagayan River. Any accretion formed by this eastern branch

which respondent Manalo may claim must be deposited on or attached to Lot 307. As it is, the claimed accretion (Lot 821) lies on

the bank of the rivernot adjacent to Lot 307 but directly opposite Lot 307 across the river.

Assuming (arguendo only) that the Cagayan River referred to in the Deeds of Sale transferring ownership of the land to respondent

Manalo is the western branch, the decision of the Court of Appeals and of the trial court are bare of factual findings to the effect

that the land purchased by respondent Manalo received alluvium from the action of the aver in a slow and gradual manner. On the

contrary, the decision of the lower court made mention of several floods that caused the land to reappear making it susceptible to

cultivation. A sudden and forceful action like that of flooding is hardly the alluvial process contemplated under Article 457 of the

Civil Code. It is the slow and hardly perceptible accumulation of soil deposits that the law grants to the riparian owner.

Besides, it is important to note that Lot 821 has an area of 11.91 hectares. Lot 821 is the northern portion of the strip of land

having a total area of 22.72 hectares. We find it difficult to suppose that such a sizable area as Lot 821 resulted from slow accretion

to another lot of almost equal size. The total landholding purchased by respondent Manalo is 10.45 hectares (8.65 hectares from

Faustina Taccad and 1.80 hectares from Gregorio Taguba in 1959 and 1964, respectively), in fact even smaller than Lot 821 which

he claims by way of accretion. The cadastral survey showing that Lot 821 has an area of 11.91 hectares was conducted in 1969. If

respondent Manalo's contention were accepted, it would mean that in a span of only ten (10) years, he had more than doubled his

landholding by what the Court of Appeals and the trial court considered as accretion. As already noted, there are steep vertical dike-like slopes separating the depressed portion or river bed and Lot 821 and Lot 307. This topography of the land, among other

things, precludes a reasonable conclusion that Lot 821 is an increment to the depressed portion by reason of the slow and constant

action of the waters of either the western or the eastern branches of the Cagayan River.

We turn finally to the issue of ownership of Lot 821. Respondent Manalo's claim over Lot 821 rests on accretion coupled with

alleged prior possession. He alleged that the parcels of land he bought separately from Gregorio Taguba and Faustina Taccad were

formerly owned by Judge Juan Taccad who was in possession thereof through his (Judge Taccad's) tenants. When ownership was

transferred to him, respondent Manalo took over the cultivation of the property and had it declared for taxation purposes in his name. When petitioners forcibly entered into his property, he twice instituted the appropriate action before the Municipal Trial Court

of Tumauini, Isabela. Against respondent Manalo's allegation of prior possession, petitioners presented tax declarations standing in

their respective names. They claimed lawful, peaceful and adverse possession of Lot 821 since 1955.

If respondent Manalo had proved prior possession, it was limited physically to Lot 307 and the depressed portion or the eastern

river bed. The testimony of Dominga Malana who was a tenant for Justina Taccad did not indicate that she was also cultivating Lot

821. In fact, the complaints for forcible entry lodged before the Municipal Trial Court of Tumauini, Isabela pertained only to Lot 307 and the depressed portion or river bed and not to Lot 821. In the same manner, the tax declarations presented by petitioners

conflict with those of respondent Manalo. Under Article 477 of the Civil Code, the plaintiff in an action for quieting of title must at

least have equitable title to or interest in the real property which is the subject matter of the action. The evidence of record on this

point is less than satisfactory and the Court feels compelled to refrain from determining the ownership and possession of Lot 821,

adjudging neither petitioners nor respondent Manalo as owner(s) thereof.

WHEREFORE, the Decision and Resolution of the Court of Appeals in CA-GR CV No. 04892 are hereby SET ASIDE. Respondent Manalo is hereby declared the owner of Lot 307. The regularly submerged portion or the eastern bed of the Cagayan River is hereby

DECLARED to be property of public dominion. The ownership of Lot 821 shall be determined in an appropriate action that may be

instituted by the interested parties inter se. No pronouncement as to costs.

SO ORDERED.

Fernan, C.J., Gutierrez, Jr., Bidin and Davide, Jr., JJ., concur.